Re: [obm-l] Limites

2002-12-06 Thread A. C. Morgado



a) Desracionalizando, dah uma fraçao cujo denominador eh    sqrt (x^2+x)
+ x   e cujo numerador eh  x.
Dividindo numerador e denominador por x, , dah  1/[sqrt (1+ 1/x) + 1]. Aih
eh facil ver que a resposta eh 1/2

b) A primeira parcela tende a  3; a segunda, a -1; a ultima, a -4.
A resposta eh 3 -1 -4 = -2

Henrique P. Sant'Anna Branco wrote:

  
  Hi all,
  
Gostaria de saber se alguém me dá uma ajuda nos seguintes  limites:
   
  a) sqrt(x^2+x)-x, com x tendendo a +infinito
  b) [[x]]-4/x-4, com x tendendo a 4 pela esquerda, onde [[x]] representa
 a função "maior inteiro"
   
  Valeu!
Henrique.
  
___
 Super iG - Internet em Alta Velocidade - http://www.superig.com.br/
  = 
Instruções para entrar na lista, sair da lista e usar a lista em http://www.mat.puc-rio.br/~nicolau/olimp/obm-l.html 
O administrador desta lista é  = 
  
  
  
  


Re: [obm-l] IME

2002-12-07 Thread A. C. Morgado



2) log(15)[ [1125]^(1/5)] =  log [1125^(1/5)] / log 15
O denominador eh  log 15 =  log(3*10/2) = log3 + log10 - log 2 =  b +1 -
a
O numerador eh   log [1125^(1/5)] = (1/5) log1125 = (1/5) log 9000/8 = (1/5)
log (9*1000/8) = 
= (1/5) [ log 9 + log1000 - log8] = (1/5) [2b+3-3a]
A resposta eh  (1+ b - a)/[5(3 + 2b - 3a)]

Wander Junior wrote:

  
  
  
1)  Encontre todas as soluções reais da equação apresentada
abaixo, onde n é um  número natural.
  
  
cosnx – sennx = 1
   
   
  2)
  Consigerando log2=a e log3=b, encontre em função de a e b o logaritmo do
número  
 no sitema de base 15.
   
   
   
  Obrigado.
  Wander
  
  
  
  


Re: [obm-l] IME

2002-12-07 Thread A. C. Morgado



1) (cosx)^n = 1 + (senx)^n
Se n eh par, o segundo membro eh maior ou igual a 1 e a igualdade so sera
possivel se senx=0, o que da as soluçoes x=k(pi)
Se n=1, a equaçao eh  cosx - senx = 1
Multiplique tudo por (sqrt2) / 2 e obtera  cos [x+(pi/4)] = cos(pi/4), que
da  x=2kpi e tambem a soluçao h = 2kpi- (pi/2)
Se n eh impar maior que 1, faça x = - z. A equaçao se transforma em  (cosz)^n
+ (senz)^n = 1. A firmo que essa equaçao so possui as soluçoes  cosz=1 e
senz=1. Basta observar que  (cosz)^2 + (senz)^2 = 1 e que se cosz e senz
estiverem no aberto (0, 1), (cosz)^n + (senz)^n sera estritamente menor que
(cosz)^2 + (senz)^2 = 1. (Eh claro que soluçoes com cosz ou senz negativos,
nem pensar). Entao, as unicas soluçoes serao  z= 2kpi  e  z= 2kpi + (pi/2).
Como x = -z, 


Wander Junior wrote:

  
  
  
1)  Encontre todas as soluções reais da equação apresentada
abaixo, onde n é um  número natural.
  
  
cosnx – sennx = 1
   
   
  2)
  Consigerando log2=a e log3=b, encontre em função de a e b o logaritmo do
número  
 no sitema de base 15.
   
   
   
  Obrigado.
  Wander
  
  
  
  


Re: [obm-l] probabilidade de novo?

2002-12-08 Thread A. C. Morgado
Emocionante! Mas vou dizer uma coisa: se você apresentar esta questão a 
tres eststisticos de diversas escolas, voce vai obter quatro respostas 
diferentes.
Morgado

Diego wrote:

Vocês devem estar cansaaados de tantas questões sobre probabilidade. Essa
surgiu num chat de mercado financeiro - uma polêmica com um cara que é muito
bem-sucedido no que faz, mas me parece ter escorregado no conceitual.

A narrativa dele é mais ou menos a seguinte. Um mendigo que nunca soube nada
de mercado abre um jornal, escolhe uma ação ao acaso e decide comprá-la. Se
o preço da ação subir, ele ganha. Se o preço da ação cair, ele perde. Qual é
a probabilidade de que um mendigo ganhe no mercado acionário?

A interpretação desse meu amigo era a seguinte: os eventos são dois: alta ou
baixa. Portanto, as chances dele ganhar são de 50%.

A minha interpretação era de que assumindo como simplificação de que cada
uma dessas ações já está "predestinada" a subir ou cair no dia seguinte -
sem que uma pequena compra do mendigo consiga influenciar o mercado como um
todo - a probabilidade de que ele ganhasse era de (número de ações
vencedoras)/(número total de ações).

Pois bem, nós estamos precisando de arbitragem... quem está certo?







=
Instruções para entrar na lista, sair da lista e usar a lista em
http://www.mat.puc-rio.br/~nicolau/olimp/obm-l.html
O administrador desta lista é <[EMAIL PROTECTED]>
=





=
Instruções para entrar na lista, sair da lista e usar a lista em
http://www.mat.puc-rio.br/~nicolau/olimp/obm-l.html
O administrador desta lista é <[EMAIL PROTECTED]>
=



Re: [obm-l] Limites

2002-12-08 Thread A. C. Morgado
Opa, muito cuidado! O limite eh igual a 1/2 e nao igual a zero.
O problema na soluçao abaixo eh o mesmo que permitiria "provar" que lim 
x = 0 (com x tendendo a mais infinito).
lim x = lim [(x^2+x) - (x^2) ] = lim [x^2(1+1/x) -x^2] entao temos  lim 
[x^2.1 - x^2] = 0.
Evidentemente, nao ha nada que valide o que estah depois do entao temos, 
tanto nesta soluçao como na soluçao abaixo.
Para uma soluçao correta vejam a enviada pelo Leandro Recova.
Morgado

Marcos Reynaldo wrote:

Opa , cuidado!
O limite da zero e não meio. Se tiver duvidas vai
jogando valores cada vez maiores e vera o que estou
dizendo. O negocio eh o seguinte (lim é o limite com x
tendendo a mais infinito)
lim(sqrt(x^2+x)-x)=lim(sqrt(x^2(1+1/x)-x)=lim(sqrt(x^2).sqrt(1+1/x)-x)
O problema eh que (sqrt(x^2))=|x|=x (se x tendesse a
menos infinito seria -x). 
entao temos
lim(x.sqrt(1+1/x)-x)=lim(x.1-x)=0

[]'s Marcos.

___
Busca Yahoo!
O melhor lugar para encontrar tudo o que você procura na Internet
http://br.busca.yahoo.com/
=
Instruções para entrar na lista, sair da lista e usar a lista em
http://www.mat.puc-rio.br/~nicolau/olimp/obm-l.html
O administrador desta lista é <[EMAIL PROTECTED]>
=




=
Instruções para entrar na lista, sair da lista e usar a lista em
http://www.mat.puc-rio.br/~nicolau/olimp/obm-l.html
O administrador desta lista é <[EMAIL PROTECTED]>
=



Re: [obm-l] livro raro

2002-12-10 Thread A. C. Morgado
Title: Re: [obm-l] livro raro



Se nao me engano, temos um exemplo de falta de comunicaçao. Insisto que mensagens
do tipo "onde se encontra um livro" deviam sempre vir acompanhadas da cidade
do remetente. Ja escrevi a respeito disso varias vezes (ninguem le o que
eu escrevo). O Wagner esta dando uma dica perfeita pensando que o Danieleh
carioca. So que, se nao me falha a memoria (e, dada a minha idade, ela falha
muito), o Daniel eh paranaense!
Morgado

Eduardo Wagner wrote:

  
Tem na biblioteca do IMPA.
  
 --
 From: "Daniel Pini" <[EMAIL PROTECTED]>
 To: <[EMAIL PROTECTED]>
 Subject: [obm-l] livro raro
 Date: Tue, Dec 10, 2002, 3:09 PM
  
  
  Alguém aqui tem ou sabe quem
tem o livro do POGORELOV, A.V. Geometria Elemental. Moscou, Mir, 1974.
 








Re: [obm-l] Probabilidade

2002-12-13 Thread A. C. Morgado
Por isso tem tanta gente que acha probabilidade complicado! Que 
enunciado mal redigido!
Quantos são os salgadinhos de cada tipo?
Admitindo-se um bando de coisas que nao estao no enunciado, talvez a 
resposta seja C(10,2)/CR(10,4) = 45/715 = 9/143


fnicks wrote:

Olá  Pessoal ,
Preciso  de  ajuda  no  seguinte  problema :

Numa  festa  há  10 tipos  de  salgadinhos( croquete, bolinho  de 
bacalhau , etc ...)  numa  mesa . Uma  pessoa  servindo -se  , pega  4 
salgadinhos num prato ; qual  a probabilidade de no prato ter dois  
pares  de salgadinhos ? ( por exemplo ; croquete , croquete , bolinho 
de carne , bolinho de carne ).

Agradeço  a  ajuda
[]´s  Nick


=
Instruções para entrar na lista, sair da lista e usar a lista em
http://www.mat.puc-rio.br/~nicolau/olimp/obm-l.html
O administrador desta lista é <[EMAIL PROTECTED]>
=




=
Instruções para entrar na lista, sair da lista e usar a lista em
http://www.mat.puc-rio.br/~nicolau/olimp/obm-l.html
O administrador desta lista é <[EMAIL PROTECTED]>
=



Re: [obm-l] Probabilidade

2002-12-14 Thread A. C. Morgado



Eh uma das interpretaçoes possiveis. Na realidade, nessa sua soluçao esta
implicito que ha um numero infinito de salgadinhos de cada tipo. Realmente,
quando voce faz casos favoraveis/ casos possiveis , voce esta adotando o
modelo equiprobabilistico, isto eh, voce admite que os casos possiveis sao
igualmente provaveis. No caso do problema dos salgadinhos, isso equivale
a supor que quando a pessoa vai pegar um salgadinho ha uma prob. 1/10 de
que seja um croquete, 1/10 de que seja um quibe etc.
Na sua soluçao, o que voce faz implicitamente eh  que a prob. de que seja
croquete croquete quibe quibe nessa ordem eh 1/10 vezes 1/10 etc. Considerando
as outras ordens, dah  4!/(2!2!) vezes isso. Finalmente, considerando que
podia ser pastel e empada em vez de croquete e quibe etc, dah C(10,2) vezes
isso. 
Agora, bem: o que voce estah supondo para fazer isso? Se ha N salgadinhos
e 10% sao croquetes, 10% sao quibes, etc (veja que isso faz com que os casos
sejam igualmente provaveis, porem isso ja eh um super acrescimo ao
enunciado), a prob. de o primeiro salgadinho ser um croquete eh
 10%N / N = 1/10, mas, depois disso, a prob. de o segundo ser croquete ja
nao eh 1/10 e sim  (0,1 N - 1) / (N-1), que so no caso N infinito dah 10%.
A sua soluçao estah absolutamente correta, DESDE QUE SE SUPONHA QUANTIDADES
IGUAIS DE SALGADINHOS DE CADA TIPO, NUMERO TOTAL DE SALGADINHOS MUITO GRANDE
E QUE O PROCESSO DE ESCOLHA SEJA O QUE VOCE DESCREVEU (pega um, depois pega
outro,...)
Como no enunciado nao havia referencia ao numero de salgadinhos de cada tipo,
preferi imaginar a seguite situaçao: o sujeito indeciso queria pegar 4 salgadinhos
e nao se decidia. Entao o garçom fez uma lista de todas as possibilidades:
4 croquetes, 3 croquetes e 1 quibe, 1 pastel e 1 quibe e 1 empada e 1 enroladinho,
etc. Dessa lista sorteou-se uma possibilidade. O numero de elementos da lista
eh combinaçao com repetiçao de 10 tomada 4 a 4, e o numero de casos favoraveis
(dois pares) eh C(10,2). dirah voce que eh preciso muita imaginaçao para
usar esse processo de escolha. Concordo que eh um processo muito artificial,
MAS FOI O UNICO QUE ME PARECEU MAIS OU MENOS PLAUSIVEL FACE A FALTA DE INFORMAÇAO
A RESPEITO  DA QUANTIDADE DE SALGADINHOS DE CADA TIPO.
Concluindo, como havia dito, talvez a resposta seja a minha, talvez seja
a sua, talvez seja uma outra. O fato eh que para se obter alguma soluçao
eh preciso supor coisas que nao estao no enunciado.
Um forte abraço
Morgado



fnicks wrote:

  

  
Professor  Morgado ,O
enunciado  foi passado realmente desta  forma  para mim .Gostaria de saber
o que  

 está  errado  na seguinte idéia : casos  possíveis =10x10x10x10(  escolhendo 
primeiro um salgadinho , depois  um segundo salgadinho e assim por diante)
; casos  favoráveis = C(10,2)X 4!/2!2! = 

 270   ou seja p = 27/1000 .

 Agradeço  novamente  desde já 

 []´s  Nick 




 To: [EMAIL PROTECTED] Subject: Re: [obm-l] Probabilidade
  

  
 Por isso tem tanta gente que acha probabilidade complicado! Que enunciado 
mal redigido! 
 Quantos são os salgadinhos de cada tipo? 
 Admitindo-se um bando de coisas que nao estao no enunciado, talvez a resposta
seja C(10,2)/CR(10,4) = 45/715 = 9/143
  
 fnicks wrote:
 Olá Pessoal , 
 Preciso de ajuda no seguinte problema :
 Numa festa há 10 tipos de salgadinhos( croquete, bolinho de bacalhau , etc
...) numa mesa . Uma pessoa servindo -se , pega 4 salgadinhos num prato ;
qual a probabilidade de no prato ter dois 
 pares de salgadinhos ? ( por exemplo ; croquete , croquete , bolinho de carne
, bolinho de carne ).
 Agradeço a ajuda 
 []´s Nick
  
  
  
  
  


Re: [obm-l] RAIZ CÚBICA DE 7

2002-12-17 Thread A. C. Morgado
Analogamente, trocando par ou impar por multiplo de 7 ou nao-multiplo de 7.

JOÃO CARLOS PAREDE wrote:


Em livros sobre conjuntos numéricos, eles quase sempre
apresentam uma prova por absurdo da irracionalidade da
raiz quadrada de 2:

sqrt(2)=p/q, sendo mdc(p,q)=1
2=(p*p)/(q*q)
2*q*q=p*p
Com isto p é par.
Analogamente se prova que q é par, caindo no absurdo.

Mas, por exemplo, com raiz cúbica de 7, como faço?

=

JOÃO CARLOS PAREDE 


___
Busca Yahoo!
O melhor lugar para encontrar tudo o que você procura na Internet
http://br.busca.yahoo.com/
=
Instruções para entrar na lista, sair da lista e usar a lista em
http://www.mat.puc-rio.br/~nicolau/olimp/obm-l.html
O administrador desta lista é <[EMAIL PROTECTED]>
=




=
Instruções para entrar na lista, sair da lista e usar a lista em
http://www.mat.puc-rio.br/~nicolau/olimp/obm-l.html
O administrador desta lista é <[EMAIL PROTECTED]>
=



Re: [obm-l] [obm-l] polinômios

2002-12-17 Thread A. C. Morgado
O Teorema de D'Alembert diz que o resto da divisao de P(x) por x+a  eh 
P( - a ).
A resposta eh P(-1) = 2

Juliana Löff wrote:

Qual é o resto da divisão de x^109 + 3 por x + 1?

=
Instruções para entrar na lista, sair da lista e usar a lista em
http://www.mat.puc-rio.br/~nicolau/olimp/obm-l.html
O administrador desta lista é <[EMAIL PROTECTED]>
=





=
Instruções para entrar na lista, sair da lista e usar a lista em
http://www.mat.puc-rio.br/~nicolau/olimp/obm-l.html
O administrador desta lista é <[EMAIL PROTECTED]>
=



Re: [obm-l] O mistério do 6174 e a irracionalidade de e^2

2002-12-17 Thread A. C. Morgado
Title: Help



1) O problema 1 se encontra em Ingenuity in Mathematics, de Ross Honsberger.
Decepcionantemente, a prova eh bastante braçal. Na realidade, o resultado
vale desde que os 4 digitos nao sejam todos iguais (vale para 3343, por exemplo).
A linha da prova eh que apos a primeira subtraçao so ha 98(?) resultados
possiveis, apos a segunda subtraçao um numero menor ainda,..., apos a setima
o resultado so pode ser 6174.
A passagem mais inteligente da demonstraçao eh a primeira.

Cláudio (Prática) wrote:

  
  href="" class="moz-txt-link-freetext" href="">file://C:\WINDOWS\> 
  
  
  Dois Problemas:
   
  1. Seja A um inteiro positivo < 10.000, formado por quatro  algarismos
distintos (possivelmente com o algarismo dos milhares igual a zero -  por
exemplo: 123 = 0123).
   
  Considere a seguinte sequência:
   
  X(1) = A
   
  Para cada inteiro positivo n:  X(n+1) = H(n) - L(n)
  onde:
  H(n) = número formado pelos algarismos de X(n) em ordem decrescente;
  L(n) = número formado pelos algarismos de X(n) em ordem crescente;
   
  Prove que, qualquer que seja A (formado por algarismos distintos),
 existe um índice m, tal que se k >=m então X(k) = 6174. 
   
   
  2. Existe uma demonstração simples da irracionalidade de e (base dos
 logaritmos naturais), baseada na identidade:
       infinito
  e = SOMATÓRIO  ( 1 / n! )
       n =  0
  A demostração é por contradição, onde a hipótese da racionalidade
de e leva  à conclusão de que existe um número inteiro entre 0 e 1.
   
  Problema: Como usar a mesma idéia para se demonstrar a irracionalidade
de  e^2.
   
  Agradeço antecipadamente qualquer ajuda.
   
  Um abraço,
  Claudio Buffara.
   
   
  
  
  
  


[obm-l] Geometria II

2002-12-17 Thread A. C. Morgado
Varias pessoas tem perguntado onde encontrar o livro Geometria II (do 
Eduardo Wagner, embora seja conhecido como o livro de Geometria do 
Morgado), que estava esgotado e era muito procurado por candidatos a 
IME, ITA etc.
O livro foi reeditado. O telefone da editora eh (21) 2581 2873.
No momento estah disponivel na editora (Rua Carneiro Ribeiro 22 loja A, 
Maria da Graça, Rio de Janeiro, RJ) ou pelo correio.
Semana que vem deve ser encontrado na Academia do Saber (Avenida Passos 
22, Rio de Janeiro) e na Ciencia e Cultura (Rua Sete de Setembro 127, 
Rio de Janeiro).
Em janeiro, ja deve ser encontrado nas livrarias do Brasil inteiro).
Morgado

=
Instruções para entrar na lista, sair da lista e usar a lista em
http://www.mat.puc-rio.br/~nicolau/olimp/obm-l.html
O administrador desta lista é <[EMAIL PROTECTED]>
=


Re: [obm-l] probabilidade

2002-12-17 Thread A. C. Morgado



A prob. de ser lata de atum eh
(1/2).(3/10)+(1/2).[x/(x+6)]
Fazendo isso ser maior que ou igual a 1/2 da  x maior que ou igual a 14.

Eder wrote:

  
  
  Por favor,ajudem-me com esta:
   
   
  Em um balcão de supermercado, foram esquecidas
duas  sacolas.Uma continha  3 latas de atum,2 de ervilha e 5 de sardinha;a
 outra,x latas de atum,3 de ervilha e 3 de sardinha.Escolhe-se
 ao acaso uma sacola e retira-se uma lata.Qual é o menor valor de 
x  para que a probabilidade de tratar-se de uma lata de atum seja,no
 mínimo, 50%?
  
   
   
  Eder
  
  
  
  


Re: [obm-l] [obm-l] polinômios

2002-12-18 Thread A. C. Morgado



A menos de notaçao, eh isso aih.
O resto eh uma constante, porque o divisor eh de primeiro grau. Chama-lo
de r estah legal.
O quociente, como nao eh necessariamente constante, deveria ter sido chamado
de q(x).A equaçao  
P(-a) = (-a+a)*q + r ficaria P(-a) = (-a+a)*q(-a) + r.

Eduardo Estrada wrote:

  Olá, sou novo na lista, mas estive pensando numa demonstração para esse
Teorema de D'alambert, o qual o prof. Morgado utilizou num e-mail anterior.
  Temos P(x) dividido por x+a. Pelo algoritmo de Euclides, vem:
  P(x) = (x+a)*q + r, onde q é o quociente da divisão e r é o resto. Então,
tomando    x = -a, vem P(-a) = (-a+a)*q + r = 0*q + r = r. Logo, de fato,
o resto da divisão é  P(-a).
  Se tiver alguma falha, por favor, peço que me corrijam.
  Abraço,
  Eduardo Estrada
  
  
  Busca Yahoo! 
  
 O melhor lugar para encontrar tudo o que você procura na Internet 
  
  
  


Re: [obm-l] Probabilidade

2002-12-18 Thread A. C. Morgado
Title: Help



Urna 1: uma bola branca
Urna 2: as outras 19 bolas.

Cláudio (Prática) wrote:

  
  href="" class="moz-txt-link-freetext" href="">file://C:\WINDOWS\> 
  
  
  Existem 10 bolas brancas (indistinguíveis) e 10 bolas pretas (idem),
as  quais devem ser distribuídas entre duas sacolas de forma que em cada
sacola haja  pelo menos uma bola.
   
  Escolhe-se uma sacola ao acaso e dela retira-se uma bola.
   
  Como deve ser a distribuição de bolas entre as duas sacolas de forma 
 a maximizar a probabilidade de a bola retirada ser branca?
  
  
  
  


Re: [obm-l] Re: [obm-l] G. Analítica - Área de Polígonos

2002-12-19 Thread A. C. Morgado





Igor GomeZZ wrote:

  Em 19/12/2002, 11:31, Josimar ([EMAIL PROTECTED]) disse:
  
O Carlos Alberto da Silva Victor escreveu um artigo a respeito desse assuntoem uma RPM, acho que há uns três anos.[]s, Josimar

A RPM tem publicação na internet tb?Não, mas há um CD com os 46(49?) primeiros numeros.Fui!### Igor GomeZZ  UIN: 29249895 Vitória, Espírito Santo, Brasil Criação: 19/12/2002 (14:34)Pare para pensar:O contrário de exercitar aimaginação é imaginar o exército.(Mário Seabra)=Instruções para entrar na lista, sair da lista e usar a lista emhttp://www.mat.puc-rio.br/~nicolau/olimp/obm-l.htmlO administrador desta lista &e
acute; <[EMAIL PROTECTED]>=






[obm-l] Re:

2002-12-19 Thread A. C. Morgado



1) < significa menor ou igual.
1 = (1/2) bcsenA < (1/2)bc < (1/2)bb
2 < bb
CQD

Eder wrote:

  
  
  Gostaria da ajuda de vcs nestes problemas
 russos:
   
   
  1)Um triângulo tem área 1 e lados a >
= b  > = c.Prove que b² > = 2.
   
  2)Defina p(x)=ax²+bx+c.Se p(x)=x não tem
nenhuma  raiz real, prove que p(p(x)) = 0 também não tem nenhuma raiz real.
   
  Grato pela ajuda.
   
  Eder
  
  
  
  


[obm-l] Re:

2002-12-19 Thread A. C. Morgado



2) Tenho a impressao de que isso eh falso. Experimentei  p(x) = x^2 +3x+2
e a equaçao p(p(x))=0 parece ter (se nao errei contas) uma raiz real entre
 -1  e  0.

Eder wrote:

  
  
  Gostaria da ajuda de vcs nestes problemas
 russos:
   
   
  1)Um triângulo tem área 1 e lados a >
= b  > = c.Prove que b² > = 2.
   
  2)Defina p(x)=ax²+bx+c.Se p(x)=x não tem
nenhuma  raiz real, prove que p(p(x)) = 0 também não tem nenhuma raiz real.
   
  Grato pela ajuda.
   
  Eder
  
  
  
  


Re: [obm-l] Re:

2002-12-20 Thread A. C. Morgado





Wagner wrote:

  
  
  
  Oi pessoal !
   
  2)Vou supor que a,b,c,x sejam números
reais e que a  é diferente de zero.
   Prove que se p(x)=x não tem nenhuma raiz
 real, então o módulo da ordenada do máximo ou do mínimo de  f(x)=p(p(x))
é maior que o módulo da ordenada do máximo ou do mínimo de  g(x)=p(x) -x
e depois prove que o sinal da derivada de segunda ordem de  f(x)=p(p(x))
e de g(x)=p(x) -x é o mesmo, assim se a segunda função não tem raiz  real
a primeira também não tem.
   
  Prova: Primeiro vou provar a segunda  hipótese: g
'' (x) =2a  ;  f(x)= a(ax^2 +bx +c)^2 +b(ax^2 +bx +c)  +c => 
  f ' (x) =2a(ax^2 +bx +c)(2ax +b) +b(2ax
+b) => f  '' (x) =4(a^2)(ax^2 +bx +c) +2a(2ax +b)^2 +2ab.
  Se a segunda hipótese é verdadeira então
f '' (x)/g  '' (x) > 0 => 2a(ax^2 +bx +c) +(2ax +b)^2 +b > 0 =>
 
  2(a^2)(x^2) +2abx +2ac + 4(a^2)(x^2) +4abx
+b^2 +b  > 0  => h(x) = 6(a^2)(x^2) +6abx +b^2 +2ac +b  > 0.
  Como o coeficiente dominante de h(x) é
positivo,  devemos apenas provar que h(x) não possui raízes reais.
  Se h(x) não possui raízes reais então
:   36(a^2)(b^2) -24{(a^2)(b^2) + 2(a^3)c + (a^2)b} < 0 =>
  12(a^2)(b^2) -48(a^3)c -24(a^2)b <
0 => 12b^2  -48ac -24b <0 => b^2 -4ac -2b < 0 => b^2-4ac <
2b ( 1  )
   
  Para provar ( 1 ) vou fazer algumas  considerações:
  Devemos ter que p(x)=x não tem raízes
reais. Logo  (b-1)^2 -4ac < 0  => b^2 -2b +1 -4ac < 0 => b^2
-4ac <  2b -1,
  logo ( 1 ) é verdadeira se p(x) = x não
possui  raízes reais CQD.
   
  Devemos provar agora a primeira hipótese.
g ' (x) =  0 => 2ax +b-1 =0 => x = (1-b)/2a => g ((1-b)/2a) =((b^2-2b+1)/4a)
 +(-b^2/2a) +c =
  =c +(-b^2-2b+1)/4a = (4ac -b^2-2b+1)/4a
 =>
  
  módulo da ordenada de máximo ou mínimo
de g (x) é |  {-(b^2+2b-1-4ac)/(4a)} | = y
  f ' (x) = 2a(ax^2 +bx +c)(2ax+b) +b(2ax
 +b) => f ' (x) = (2ax +b)(2(a^2)(x^2) +2abx +2ac  +b)   ; f ' (x) =0
=>
  (2ax +b) =0 ou (2(a^2)(x^2) +2abx +2ac
+b)  =0.
  O primeiro caso implica em: x= -b/2a
  O segundo caso implica em: delta= 4(a^2)(b^2)
 -4(4(a^3)c + 2(a^2)b).
  Vamos provar que delta < 0 :   4(a^2)(b^2)
-4(4(a^3)c +2(a^2)b) < 0 => b^2 -4ac -2b < 0 => b^2-4ac  <
2b ( 1 ).
  Como ( 1 ) já foi provado, então ficamos
só com o  caso x= -b/2a =>
  f(-b/2a) = a((b^2/4a) -(b^2/2a) +c)^2
+b((b^2/4a)  -(b^2/2a) +c) +c = a(c -(b^2/4a))^2 +b(c -(b^2/4a)) +c =
  =a{c^2 -c(b^2)/2a +(b^4/16a^2)}+b(c -(b^2/4a))
+c =  a(c^2) -c(b^2)/2 +b^4/16a +bc -b^3/4a +c =>
  módulo da ordenada de máximo ou mínimo
de f (x) é |  {a(c^2) -c(b^2)/2 +b^4/16a +bc -b^3/4a +c} | = z.
  Como a segunda hipótese é verdadeira então
se g(x)  tem máximo definido f(x) também tem, e se g(x)
  tem mínimo definido f(x) também tem. Temos
que se  p(x) =x não tem raiz real f '(x) e g'(x) só tem uma
  raiz real, note que se a > 0, g(x)
tem mínimo e  se a < 0, g(x) tem máximo. Logo para provar a primeira hipótese,
 temos
  que considerar 2 casos : a > 0 e a
<  0.
  Suponha que a primeira hipótese seja  falsa:
  a > 0 => y > z e y,z >  0 =>
g((1-b)/2a) > f(-b/2a) => -b^2/4a -b/2a +1/4a  +c > a(c^2) -c(b^2)/2
+b^4/16a +bc -b^3/4a +c =>
  -4b^2 -8b +4 > 16(a^2)(c^2) -8ac(b^2)
+b^4 +16bc  -4b^3 => 16(a^2)(c^2) -8ac(b^2) +b^4 +16bc -4b^3 +4b^2 -8b
+4 =h(a) <  0
  Considere ( 2 ) uma função do 2º grau
de variável  a. Temos a > 0, logo:
  64(b^4)(c^2) -64(b^4)(c^2) -64(c^2)(16bc
-4b^3  +4b^2 -8b +4) < 0 => 16bc -4b^3 +4b^2 -8b +4> 0 ( 3 ).
  De ( 2 ) vem que: (b^2 -4ac)^2 < -(16bc
 -4b^3 +4b^2 -8b +4) < 0 . Absurdo !
  Para o caso a < 0 => y > z, temos
um  raciocínio análogo, provamos que se a < 0, então h(a) > 0, logo
o delta de  h(a)
  é negativo, o que nos leva a conclusão
de que (b^2  -4ac)^2 < 0 Absurdo !
  Logo a primeira hipótese é verdadeira,
porque é  absurdo que ela seja falsa se a segunda hipótese é verdadeira,
  Logo p(x)=x não ter raízes reais implica
na segunda  hipótese qua implica na primeira.
  Se a primeira e a segunda hipóteses são
ambas  verdadeiras, isso implica que p(p(x))=0 não tem nenhuma raiz real
  CQD.
   Isso eh falso. Se  p(x) = x^2 +3x+2, a equaçao p(p(x))=0 tem 
uma raiz real entre  -1  e  0.
  
  
  OBS:Me desculpem pelo e-mail que eu mandei
sem  querer antes, ele estava com a resposta pela metade.
   
   
  André T.
   
  
  
  

- Original Message - 

From:
Eder


To:
[EMAIL PROTECTED]


Sent: Thursday, December 19, 2002 5:32PM


Gostaria da ajuda de vcs nestes problemas
   russos:
 
 
1)Um triângulo tem área 1 e lados a
> = b> = c.Prove que b² > = 2.
 
2)Defina p(x)=ax²+bx+c.Se p(x)=x não
tem nenhumaraiz real, prove que p(p(x)) = 0 também não tem nenhuma raiz
 real.
 
Grato pela ajuda.
 
Eder







Re: [obm-l] Re:

2002-12-20 Thread A. C. Morgado





Wagner wrote:

  
  
  Oi pessoal !
   
  2)Vou supor que a,b,c,x sejam números
 reais.
   Prove que o módulo da ordenada do máximo
ou  do mínimo de f(x)=p(p(x)) é maior que o módulo da ordenada do máximo
ou do  mínimo de g(x)=p(x) -x e depois prove que o sinal da derivada de segunda
ordem  de f(x)=p(p(x)) e de g(x)=p(x) -x é o mesmo, assim se a segunda função
não tem  raiz real a primeira também não tem.
   
  Prova: Primeiro vou provar a segunda  hipótese: g
'' (x) =2a  ;  f(x)= a(ax^2 +bx +c)^2 +b(ax^2 +bx +c)  +c => 
  f ' (x) =2a(ax^2 +bx +c)(2ax +b) +b(2ax
+b) => f  '' (x) =4(a^2)(ax^2 +bx +c) +2a(2ax +b)^2 +2ab.
  Se a segunda hipótese é verdadeira então
f '' (x)/g  '' (x) > 0 => 2a(ax^2 +bx +c) +(2ax +b)^2 +b > 0 =>
 
  2(a^2)(x^2) +2abx +2ac + 4(a^2)(x^2) +4abx
+b^2 +b  > 0  => h(x) = 6(a^2)(x^2) +6abx +b^2 +2ac +b  > 0.
  Como o coeficiente dominante de h(x) é
positivo,  devemos apenas provar que h(x) não possui raízes reais.
  Se h(x) não possui raízes reais então
:   36(a^2)(b^2) -24{(a^2)(b^2) + 2(a^3)c + (a^2)b} < 0 =>
  12(a^2)(b^2) -48(a^3)c -24(a^2)b <
0 => 12b^2  -48ac -24b <0 => b^2 -4ac -2b < 0 => b^2-4ac <
2b ( 1  )
   
  Para provar ( 1 ) vou fazer algumas  considerações:
  Devemos ter que p(x)=x não tem raízes
reais. Logo  (b-1)^2 -4ac < 0  => b^2 -2b +1 -4ac < 0 => b^2
-4ac <  2b -1,
  logo ( 1 ) é verdadeira se p(x) = x não
possui  raízes reais CQD.
   
  Devemos provar agora a primeira hipótese.
g ' (x) =  0 => 2ax +b-1 =0 => x = (1-b)/2a => g ((1-b)/2a) =((b^2-2b+1)/4a)
 +(-b^2/2a) +c =
  =c +(-b^2-2b+1)/4a = (4ac -b^2)/4a  =>
  
  módulo da ordenada de máximo ou mínimo
de g (x) é  |(b^2 -4ac)/(4a)|
  f ' (x) = 2a(ax^2 +bx +c)(2ax+b) +b(2ax
 +b) => f ' (x) = (2ax +b)(2(a^2)(x^2) +2abx +2ac  +b)   ; f ' (x) =0
=>
  (2ax +b) =0 ou (2(a^2)(x^2) +2abx +2ac
+b)  =0.
  O primeiro caso implica em x= -b/2a
   
   Isso eh falso. Se  p(x) = x^2 +3x+2,  a equaçao p(p(x))=0 tem
uma raiz real entre  -1  e  0.
  
  
   
  
   
  

- Original Message - 

From:
Eder


To:
[EMAIL PROTECTED]


Sent: Thursday, December 19, 2002 5:32PM


Gostaria da ajuda de vcs nestes problemas
   russos:
 
 
1)Um triângulo tem área 1 e lados a
> = b> = c.Prove que b² > = 2.
 
2)Defina p(x)=ax²+bx+c.Se p(x)=x não
tem nenhumaraiz real, prove que p(p(x)) = 0 também não tem nenhuma raiz
 real.
 
Grato pela ajuda.
 
Eder







Re: [obm-l] Re:

2002-12-20 Thread A. C. Morgado





Wagner wrote:

  
  
   
  

- Original Message - 

From:Wagner


To:
[EMAIL PROTECTED]


Sent: Friday, December 20, 2002 3:16PM

Subject: Re: 



Oi pessoal !
 
SÓ CORRIGINDO UMA COISA QUE FICOU CONFUSA
NAMINHA ÚLTIMA MENSAGEM
 
2)Vou supor que a,b,c,x sejam números
reais e quea é diferente de zero.
 Prove que se p(x)=x não tem nenhuma
raizreal, então o módulo da ordenada do máximo ou do mínimo def(x)=p(p(x))
é maior que o módulo da ordenada do máximo ou do mínimo deg(x)=p(x) -x
e depois prove que o sinal da derivada de segunda ordem def(x)=p(p(x))
e de g(x)=p(x) -x é o mesmo, assim se a segunda função não temraiz real
a primeira também não tem.
 
Prova: Primeiro vou provar a segunda
   hipótese: g '' (x) =2a  ;  f(x)= a(ax^2 +bx +c)^2 +b(ax^2 +bx+c) +c
=> 
f ' (x) =2a(ax^2 +bx +c)(2ax +b) +b(2ax
+b) =>f '' (x) =4(a^2)(ax^2 +bx +c) +2a(2ax +b)^2 +2ab.
Se a segunda hipótese é verdadeira então
f ''(x)/g '' (x) > 0 => 2a(ax^2 +bx +c) +(2ax +b)^2 +b > 0 =>
   
2(a^2)(x^2) +2abx +2ac + 4(a^2)(x^2)
+4abx +b^2+b > 0  => h(x) = 6(a^2)(x^2) +6abx +b^2 +2ac +b> 0.
Como o coeficiente dominante de h(x)
é positivo,devemos apenas provar que h(x) não possui raízes reais.
Se h(x) não possui raízes reais então
: 36(a^2)(b^2) -24{(a^2)(b^2) + 2(a^3)c + (a^2)b} < 0 =>
12(a^2)(b^2) -48(a^3)c -24(a^2)b <
0 =>12b^2 -48ac -24b <0 => b^2 -4ac -2b < 0 => b^2-4ac
< 2b ( 1)
 
Para provar ( 1 ) vou fazer algumas
   considerações:
Devemos ter que p(x)=x não tem raízes
reais. Logo(b-1)^2 -4ac < 0  => b^2 -2b +1 -4ac < 0 => b^2
   -4ac < 2b -1,
logo ( 1 ) é verdadeira se p(x) = x
não possuiraízes reais CQD.
 
Devemos provar agora a primeira hipótese.
g ' (x)= 0 => 2ax +b-1 =0 => x = (1-b)/2a => g ((1-b)/2a) =((b^2-2b+1)/4a)
   +(-b^2/2a) +c =
=c +(-b^2-2b+1)/4a = (4ac -b^2-2b+1)/4a
   =>

módulo da ordenada de máximo ou mínimo
de g (x) é| {-(b^2+2b-1-4ac)/(4a)} | = y
f ' (x) = 2a(ax^2 +bx +c)(2ax+b) +b(2ax
   +b) => f ' (x) = (2ax +b)(2(a^2)(x^2) +2abx +2ac+b)   ; f ' (x)
=0 =>
(2ax +b) =0 ou (2(a^2)(x^2) +2abx +2ac
+b)=0.
O primeiro caso implica em: x= -b/2a
O segundo caso implica em: delta= 4(a^2)(b^2)
   -4(4(a^3)c + 2(a^2)b).
Vamos provar que delta < 0 :    
4(a^2)(b^2) -4(4(a^3)c +2(a^2)b) < 0 => b^2 -4ac -2b < 0 =>  
 b^2-4ac < 2b ( 1 ).
Como ( 1 ) já foi provado, então ficamos
só com ocaso x= -b/2a =>
f(-b/2a) = a((b^2/4a) -(b^2/2a) +c)^2
+b((b^2/4a)-(b^2/2a) +c) +c = a(c -(b^2/4a))^2 +b(c -(b^2/4a)) +c =
=a{c^2 -c(b^2)/2a +(b^4/16a^2)}+b(c
-(b^2/4a)) +c= a(c^2) -c(b^2)/2 +b^4/16a +bc -b^3/4a +c =>
módulo da ordenada de máximo ou mínimo
de f (x) é| {a(c^2) -c(b^2)/2 +b^4/16a +bc -b^3/4a +c} | = z.
Como a segunda hipótese é verdadeira
então seg(x) tem máximo definido f(x) também tem, e se g(x)
tem mínimo definido f(x) também tem.
Temos que sep(x) =x não tem raiz real f '(x) e g'(x) só tem uma
raiz real, note que se a > 0, g(x)
tem mínimoe se a < 0, g(x) tem máximo. Logo para provar a primeira
hipótese,temos
que considerar 2 casos : a > 0 e
a <0.
Suponha que a primeira hipótese seja
   falsa:
a > 0 => y > z e y,z >  
 0 => g((1-b)/2a) > f(-b/2a) => -b^2/4a -b/2a +1/4a+c > a(c^2)
-c(b^2)/2 +b^4/16a +bc -b^3/4a +c =>
-4b^2 -8b +4 > 16(a^2)(c^2) -8ac(b^2)
+b^4+16bc -4b^3 => 16(a^2)(c^2) -8ac(b^2) +b^4 +16bc -4b^3 +4b^2 -8b
+4 =h(a)< 0 ( 2 )
Considere h(a) uma função do 2º grau
de variávela. Temos a > 0, logo:
64(b^4)(c^2) -64(b^4)(c^2) -64(c^2)(16bc
-4b^3+4b^2 -8b +4) < 0 => 16bc -4b^3 +4b^2 -8b +4> 0 ( 3 ).
De ( 2 ) vem que: (b^2 -4ac)^2 <
-(16bc-4b^3 +4b^2 -8b +4) < 0 . Absurdo !
Para o caso a < 0 => y > z,
temos umraciocínio análogo, provamos que se a < 0, então h(a) >
0, logo o deltade h(a)
é negativo, o que nos leva a conclusão
de que(b^2 -4ac)^2 < 0 Absurdo !
Logo a primeira hipótese é verdadeira,
porque éabsurdo que ela seja falsa se a segunda hipótese é verdadeira,
Logo p(x)=x não ter raízes reais implica
nasegunda hipótese qua implica na primeira.
Se a primeira e a segunda hipóteses
são ambasverdadeiras, isso implica que p(p(x))=0 não tem nenhuma raiz
real
CQD.
Isso eh falso. Se  p(x) = x^2 +3x+2, a equaçao p(p(x))=0 tem 
uma raiz real entre  -1  e  0.


OBS:Me desculpem pelo e-mail que eu
mandei semquerer antes, ele estava com a resposta pela metade.
 
 
André T.
 



  
- Original Message - 
  
From:
Eder
  
  
To:
[EMAIL PROTECTED]
  
  
Sent: Thursday, December 19, 2002 5:32  PM
  
  
  Gostaria da ajuda de vcs nestes problemas
 russos:
   
   
  1)Um triângulo tem área 1 e lados
a > =  b > = c

Re: [obm-l] Re:

2002-12-20 Thread A. C. Morgado



Vou tentar encerrar a discussao. Tome p(x) = x^2 + 4x + 3. A equaçao p(x)
= x reduz-se a  x^2 + 3x + 3 = 0 ue nao tem raiz real pois seu discriminante
eh negativo (-3). Como p(-2) = -1, p(p(-2)) = p(-1) = 0, NAO EH VERDADE que
p(p(x))=0 nao possua raiz real, pois -2 eh raiz da referida equaçao. Assim
como esse, ha muitos contraexemplos que podem ser dados (vejam mensagem de
Salvador Addas Zanata).
Peço desculpas a todos pelo contraexemplo que mandei em mensagens anteriores,
pois ele estah errado.
Morgado 

Eder wrote:

  
  Esse problema foi retirado do site do
John Scholes  e o enunciado é:
   
  Define p(x)=ax²+bx+c.If p(x)=x has no
real  roots,prove that p(p(x))=0 has no real roots.
   
  

- Original Message - 

From:
A. C.Morgado


To:
[EMAIL PROTECTED]


Sent: Friday, December 20, 2002 5:12PM

Subject: Re: [obm-l] Re:




Wagner wrote:

  
  
  
  Oi pessoal !
   
  2)Vou supor que a,b,c,x sejam números
reais e  que a é diferente de zero.
   Prove que se p(x)=x não tem nenhuma
raiz  real, então o módulo da ordenada do máximo ou do mínimo de
 f(x)=p(p(x)) é maior que o módulo da ordenada do máximo ou do mínimo de
 g(x)=p(x) -x e depois prove que o sinal da derivada de segunda ordem
de  f(x)=p(p(x)) e de g(x)=p(x) -x é o mesmo, assim se a segunda função
não tem  raiz real a primeira também não tem.
   
  Prova: Primeiro vou provar a segunda
 hipótese: g '' (x) =2a  ;  f(x)= a(ax^2 +bx +c)^2 +b(ax^2 +bx  +c)
+c => 
  f ' (x) =2a(ax^2 +bx +c)(2ax +b) +b(2ax
+b)  => f '' (x) =4(a^2)(ax^2 +bx +c) +2a(2ax +b)^2 +2ab.
  Se a segunda hipótese é verdadeira
então f ''  (x)/g '' (x) > 0 => 2a(ax^2 +bx +c) +(2ax +b)^2 +b
> 0 =>  
  2(a^2)(x^2) +2abx +2ac + 4(a^2)(x^2)
+4abx +b^2  +b > 0  => h(x) = 6(a^2)(x^2) +6abx +b^2 +2ac +b  
   > 0.
  Como o coeficiente dominante de h(x)
é  positivo, devemos apenas provar que h(x) não possui raízes  reais.
  Se h(x) não possui raízes reais então
:   36(a^2)(b^2) -24{(a^2)(b^2) + 2(a^3)c + (a^2)b} < 0  =>
  12(a^2)(b^2) -48(a^3)c -24(a^2)b <
0 =>  12b^2 -48ac -24b <0 => b^2 -4ac -2b < 0 => b^2-4ac
< 2b ( 1  )
   
  Para provar ( 1 ) vou fazer algumas
 considerações:
  Devemos ter que p(x)=x não tem raízes
reais.  Logo (b-1)^2 -4ac < 0  => b^2 -2b +1 -4ac < 0 =>
b^2  -4ac < 2b -1,
  logo ( 1 ) é verdadeira se p(x) =
x não possui  raízes reais CQD.
   
  Devemos provar agora a primeira hipótese.
g '  (x) = 0 => 2ax +b-1 =0 => x = (1-b)/2a => g ((1-b)/2a)
 =((b^2-2b+1)/4a) +(-b^2/2a) +c =
  =c +(-b^2-2b+1)/4a = (4ac -b^2-2b+1)/4a
 =>
  
  módulo da ordenada de máximo ou mínimo
de g (x)  é | {-(b^2+2b-1-4ac)/(4a)} | = y
  f ' (x) = 2a(ax^2 +bx +c)(2ax+b) +b(2ax
 +b) => f ' (x) = (2ax +b)(2(a^2)(x^2) +2abx +2ac  +b)   ; f '
(x) =0 =>
  (2ax +b) =0 ou (2(a^2)(x^2) +2abx
+2ac +b)  =0.
  O primeiro caso implica em: x=   
  -b/2a
  O segundo caso implica em: delta=
4(a^2)(b^2)  -4(4(a^3)c + 2(a^2)b).
  Vamos provar que delta < 0 :   
   4(a^2)(b^2) -4(4(a^3)c +2(a^2)b) < 0 => b^2 -4ac -2b < 0 =>
 b^2-4ac < 2b ( 1 ).
  Como ( 1 ) já foi provado, então ficamos
só com  o caso x= -b/2a =>
  f(-b/2a) = a((b^2/4a) -(b^2/2a) +c)^2
 +b((b^2/4a) -(b^2/2a) +c) +c = a(c -(b^2/4a))^2 +b(c -(b^2/4a)) +c  
   =
  =a{c^2 -c(b^2)/2a +(b^4/16a^2)}+b(c
-(b^2/4a))  +c = a(c^2) -c(b^2)/2 +b^4/16a +bc -b^3/4a +c =>
  módulo da ordenada de máximo ou mínimo
de f (x)  é | {a(c^2) -c(b^2)/2 +b^4/16a +bc -b^3/4a +c} | = z.
  Como a segunda hipótese é verdadeira
então se  g(x) tem máximo definido f(x) também tem, e se g(x)
  tem mínimo definido f(x) também tem.
Temos que  se p(x) =x não tem raiz real f '(x) e g'(x) só tem uma
  raiz real, note que se a > 0, g(x)
tem  mínimo e se a < 0, g(x) tem máximo. Logo para provar a primeira
hipótese,  temos
  que considerar 2 casos : a > 0
e a <  0.
  Suponha que a primeira hipótese seja
 falsa:
  a > 0 => y > z e y,z >
 0 => g((1-b)/2a) > f(-b/2a) => -b^2/4a -b/2a +1/4a  +c >
a(c^2) -c(b^2)/2 +b^4/16a +bc -b^3/4a +c =>
  -4b^2 -8b +4 > 16(a^2)(c^2) -8ac(b^2)
+b^4  +16bc -4b^3 => 16(a^2)(c^2) -8ac(b^2) +b^4 +16bc -4b^3 +4b^2
-8b +4 =h(a)  < 0
  Considere ( 2 ) uma função do 2º grau
de  variável a. Temos a > 0, logo:
  64(b^4)(c^2) -64(b^4)(c^2) -64(c^2)(16bc
-4b^3  +4b^2 -8b +4) < 0 => 16bc -4b^3 +4b^2 -8b +4> 0 ( 3 ).
  De ( 2 ) vem que: (b^2 -4ac)^2 <
-(16bc  -4b^3 +4b^2 -8b +4) < 0 . Absurdo

Re: [obm-l] Re:

2002-12-22 Thread A. C. Morgado



O que gerou a discussao eh que trocaram p(p(x)) = x por p(p(x))=0, o que
torna o enunciado falso.
Morgado

Pedro Antonio Santoro Salomão wrote:

  - Original Message -From: "Paulo Rodrigues" <[EMAIL PROTECTED]>To: <[EMAIL PROTECTED]>Sent: Sunday, December 22, 2002 12:45 AMSubject: [obm-l] Re:
  
Não acompanhei todas as mensagens desta discussão, mas gostaria deobservar que o problema em discussão aparece como o de número 303 -página 60 do livro "Selected Problems and Theorems  in ElementaryMathematics" da Mir:303. A quadratic trinomial p(x)=ax^2+bx+c is such that the equationp(x)=x has no real roots. Prove that in this case te equation p(p(x))=xhas no real roots either.

Esse problema tem uma solução simples, como já mencionou o Salvador. Comop(x)=x não tem solução real então p(x)>x  ou p(x)p(p(x))>p(x)>x  ou p(p(x))Isso também mostra que esse resultado vale para qualquer função contínua,não precisando ser necessariamente um polinômio.Abraço. Pedro.

  Abraços, Paulo___Busca Yahoo!O melhor lugar para encontrar tudo o que você procura na Internethttp://br.busca.yahoo.com/=Instruções para entrar na lista, sair da lista e usar a lista emhttp://www.mat.puc-rio.br/~nicolau/olimp/obm-l.htmlO administrador desta lista é <[EMAIL PROTECTED]>=Esta mensagem foi verificada pelo E-mail Protegido Terra.Scan engine: VirusScan / Atualizado em 18/12/2002 / Versão: 1.3.13
Proteja o seu e-mail Terra: http://www.emailprotegido.terra.com.br/
  
  =Instruções para entrar na lista, sair da lista e usar a lista emhttp://www.mat.puc-rio.br/~nicolau/olimp/obm-l.htmlO administrador desta lista é <[EMAIL PROTECTED]>=
  
  
  
  


Re: [obm-l] Probabilidade

2002-12-23 Thread A. C. Morgado



Estrategia:
Algum resultado eh 5  menos ( algum resultado eh 5 e os outros sao impares)

Soluçao: Algum resultado eh 5      -- prob. igual a  1 - (5/6)^3
algum resultado eh 5 e os outros sao impares
   --  5 I I  (I significa impar diferente de 5) prob. (1/6).(2/6).(2/6)
(ha outros dois casos analogos, I5I e II5)
 -- 55I  prob. (1/6)(1/6)(2/6) (ha outros dois casos analogos, 5I5 e I55)
--  555 prob. (1/6)(1/6)(1/6)
A resposta eh 1 - (5/6)^3 - 3. (1/6).(2/6).(2/6) - 3.(1/6)(1/6)(2/6)  - (1/6)(1/6)(1/6)
= 1/3

Outra estrategia: Algum dado tem que dar 5.
a) dois dados dao 5 e o outro da par. 3.(1/6).(1/6).(1/2) = 1/24
b) um dado da 5 e os dois outros dao par.  3.(1/6).(1/2) (1/2) = 1/8
c) um dado da 5, outro da par e outro da impar diferente de 5.  3!(1/6).(1/2).
(2/6) = 1/6
A resposta eh 1/24 + 1/8 + 1/6 =  1/3

Daniel wrote:

  
  
               Olá a todos, é fácil, mas
não saiu:
   
                   Jogando-se sucessecivamente
um dado de  seis faces não viciado, em lances independentes, três vezes,
 qual probabilidade do produto dos 3 números ser múltiplo de  dez?
   
  Grato
   
  Daniel
  
  
  
  


Re: [obm-l] RE: [obm-l] classifiquem a função

2003-01-13 Thread A. C. Morgado



Tirando uma palavra do ostracismo:
essas funçoes que sao quocientes de polinomios de primeiro grau  ( de raizes
diferentes ) tem como graficos hiperboles equilateras com uma assintota horizontal
e outra vertical  sao (ou eram, ha muito tempo nao vejo ninguem uar esse
termo) chamadas (pelo menos pelos franceses) de HOMOGRAFICAS.
Morgado

Alguem ha muito tempo escreveu:

  
  
  
  
  
  
Como vocês classificariam a seguinte função: 
 f(x) = x-a / bx+a. Nos livros de 2º grau temos a função afim ou do 1ºgrau
com a expressão ax+b. Temos a função quadrática com a expressão ax^2+bx+c
etc. E na expressão da função acima como poderiamos classificar ? Eu acho
que é do 1º grau pois temos somente a variável x com expoente unitário, mas
e quanto ao gráfico? 
  
  
  
  
  


Re: [obm-l] Variaveis aleatorias

2003-01-13 Thread A. C. Morgado
<= significa menor que ou igual a.
Pondo Y = X^2 e usando F para a funçao de distribuiçao de Y, temos:
F(y) = P(Y<= y) = P(X^2 < = y) = 0  se y <= 0 
F(y) = P(Y<= y) = P(X^2 < = y) = 1 se y >= 4
F(y) = P(Y<= y) = P(X^2 < = y) = P( -sqrt y < X < sqrt y) = (2/3) sqrt 
y, se  0 < y <= 1
F(y) = P(Y<= y) = P(X^2 < = y) = P( -sqrt y < X < sqrt y) = P(-1 < X < 
sqrt y) =
= (1/3). (sqrt y + 1) , se 1 < y < 2
[EMAIL PROTECTED] wrote:

Oi pessoal,
   eu estava fazendo alguns
exercicios sobre variaveis aleatorias
e empaquei neste aqui (porque do jeito
que estou fazendo está errado pois a
funcao de distribuicao pode dar maior
do que 1, o que está não está certo).
O exercicio eh o seguinte:

Seja f(x) = 1/3,  -1 < x < 2 e 0 para
quaisquer outros valores de X.
Encontre a funçao de distribuicao da
variavel aleatoria Y = X^2.

Obrigado antes de tudo,
Eduardo


__
E-mail Premium BOL
Antivírus, anti-spam e até 100 MB de espaço. Assine já!
http://email.bol.com.br/


=
Instruções para entrar na lista, sair da lista e usar a lista em
http://www.mat.puc-rio.br/~nicolau/olimp/obm-l.html
O administrador desta lista é <[EMAIL PROTECTED]>
=





=
Instruções para entrar na lista, sair da lista e usar a lista em
http://www.mat.puc-rio.br/~nicolau/olimp/obm-l.html
O administrador desta lista é <[EMAIL PROTECTED]>
=



Re: [obm-l] probabilidade

2003-01-14 Thread A. C. Morgado



A observaçao sobre Flamengo e Botafogo nao eh verdadeira se o Botafogo jogar
com a equipe reserva.
Morgado

Marcos Paulo wrote:

  
  
  Neste caso a quantidade de resultados
possíveis é  bem pequena e portanto é possível lista-los.
  Seja A a vitória da primeira equipe e
B a vitória  da segunda equipe. Os resultados possíveis são:
  ABABA
  ABAA
  AA
  BAA
  BABAA
  BABAB
  BABB
  BB
  ABB
  ABABB
   
  dentre as 10 possibilidades listadas há
8 em que  uma das equipes tem 2 vitórias consecutivas (80%).
   
  Tenho ainda uma dúvida quanto a este "uma
 equipe". Se "uma equipe" for o mesmo que "alguma equipe" então a resposta
é  a dada acima. 
  Se, por outro lado, "uma equipe" for o
mesmo que  "uma determinada equipe" então a resposta será metade da resposta
 dada.
  Para deixar mais claro o que não entendi
vou  modificar o enunciado e gostaria que me respondessem se o enunciado
é  equivalente ao pedido nesta mensagem.
  "Palmeiras e Botafogo disputam um torneio
de  futebol em que saírá vencedor aquele que obtiver 2 vitórias consecutivas
ou 3  vitórias alternadas. Se não é possível o empate (disputa de penalti),
qual a  probabilidade de que o Botafogo saia vitorioso do torneio com duas
vitórias  consecutivas?"
  Obs. A escolha dos times não foi aleatória
..  procurei times que tivessem igualdade de condições na disputa de qualquer
 partida, ou seja, a vitória de qualquer um dos times contece com 50% de
 probabilidade. Nunca iria colocar, por exemplo,  Flamengo e Botafogo visto
 que estes estão em categorias diferentes e portanto a vitória do Flamengo
 aconteceria com maior probabilidade.
  Não sei se minha dúvida procede mas resolvi
 responder apenas para tirar esta dúvida.
   
  []'s MP
  

- Original Message - 

From:[EMAIL PROTECTED]


To:
[EMAIL PROTECTED]


Sent: Monday, January 13, 2003 11:36PM

Subject: [obm-l] probabilidade



Duas equipes disputam entre si uma série de jogos em quenão pode ocorrer
empate e as duas equipes têm as mesmas chances de vitória. Aprimeira
equipe que conseguir duas vitórias seguidas ou três vitóriasalternadas
vence a série de jogos. Qual a probabilidade de uma equipe vencer asérie
de jogos com duas vitórias  seguidas?







Re: [obm-l] polinômios

2003-01-15 Thread A. C. Morgado



Voce estah certo; o gabarito, errado.
Morgado
[EMAIL PROTECTED] wrote:

Olá pessoal,  
  
Vejam a questão abaixo: 
  
(U.C. SALVADOR) O quociente da divisão do polinômio p= x^3 - 3x^2 + 3x -
1 pelo polinômio p= x - 1 é: 
  
Dúvida: Eram 5 alternativas e segunda o gabarito a resposta certa é x^2 -
3x + 3 (alternativa "e"), só que eu estou chegando somente na alternativa
"d" que diz que o quociente é x^2 - 2x +1. Eu cheguei neste resultado aplicando
o algoritmo de briot-ruffini, pois o divisor é do tipo x-a. Não está certo?
  
  
  
  
  


Re: [obm-l] figuras planas (Fuvest)

2003-01-16 Thread A. C. Morgado



S=pr
r= S/p = 6/6 = 1 cm

[EMAIL PROTECTED] wrote:

Olá pessoal,  
  
Alguém poderia me ajudar nesta questão: 
  
(FUVEST) Um triângulo tem 12 cm de perímetro e 6 cm^2 de área. Quanto mede
o raio da circunferência inscrita nesse triângulo? 
  
  
  
  
  


Re: [obm-l] Sistema de equações

2003-01-16 Thread A. C. Morgado



Para uma soma de uadrados de reais dar zero, ambos devem valer zero. Logo,
4x + 2y - 5 = 0  e   3x - y + 1 = 0.
Daí, x = 3/10 e  y = 19/10.
x+y = 22/10 = 1/5.

[EMAIL PROTECTED] wrote:

Olá pessoal, 
  
Alguém pode me ajudar nesta questão: 
  
Os números reais x e y para os quais (4x + 2y - 5)^2 + (3x - y + 1)^2 =0
são tais que x + y vale:  
  
Resp: 11/5 
  
Obs: Eu tentei produtos notáveis, mas não deu certo pois são três parcelas
nos parênteses, depois eu tentei multiplicar os parenteses mas não deu para
isolar o x e o y em um membro para eu conseguir o resultado direto ao invés
de descobrir o valor de x e depois de y. 
  
  
  
  
  


Re: [obm-l] Livro Geometria

2003-01-17 Thread A. C. Morgado
Agradecemos as referencias elogiosas do Paulo.
Corrijo as declaraçoes do Wagner:
1) o Geometria 1 eh um bom livro. Claro que, por tratar de conceitos
mais basicos e se dirigir basicamente a um leitor que seja um bom aluno
de SETIMA SERIE, sempre os autores (e leitores mais adiantados) acharao
que se poderia dar um tratamento "axiomaticamente" mais rigoroso; mas o
objetivo era fazer um livro de Geometria para vestibulares dificeis e
concursos de admissao a escolas militares, extremamente procuradas na
epoca; ainda hoje eh um bom livro para IME, ITA, Colegio Naval, etc.
2) Geometria 1, Geometria 2 e Algebra 1 podem ser pedidos a
FC&Z Livros
Rua Carneiro Ribeiro 22 loja A
21050-570  Maria da GraçaRio de JaneiroRJ
Telefax  (21) 2581-2873
Morgado

Paulo Santa Rita wrote:

> Ola Leonardo e demais
> colegas desta lista ... OBM-L,
>
> Foi esse livro que despertou meu interesse pela Matematica.
>
> Voce teve a mesma impressao que eu tive, quando o li pela primeira
> vez, apos adquiri-lo em um sebo ( sebo = livraria de livros usados ).
> Muitas vezes me perguntei o que o tornava tao interessante e diferente
> dos outros ...
>
> Me parece que e porque os autores ( Eduardo Wagner e Augusto Morgado )
> nao perdem tempo provando coisas simples e evidentes, partindo
> imediatamente a exposicao de fatoss espetaculares e inusitadas, que
> dificilmente imaginariamos que ocorrem.
>
> Quando um matematico explica um fenomeno inusitado ( por exemplo : o
> circulo de nove pontos ) ele aguca nossa inteligencia e nos vemos
> claramente que a Matemaica nao se resume a provas de fatos evidentes
> e, portanto, desmotivadoras ( por exemplo : prove que 1+1=2 ).
>
> Um Matematico adulto pode apreciar o formalismo, mas uma mente nova
> geralmente nao aprecia estas coisas ... Prove a uma crianca algo que
> ela duvida e nao suspeia e voce vai conquistar o interesse dela ... me
> parece que e esse simples detalhe que diferencia as grandes obras da
> mesmice e mediocridade que campeia na imensa maioria das obras
> didaticas da matematica ...
>
> As Olimpiadas de Matematica, que todos nos gostamos e admiramos e
> pelas quais fazemos verdadeiros sacrificios e uma continuacao de tudo
> isso ... Por que elas fazem sucesso ? Simplesmente porque as pessoas
> inteligentes odeiam coisas rotineiras e burocraticas, que sao os
> aspectos tipicos do ensino comum ...
>
> Neste sentido, o Livro a que voce se refere, o Geometria II dos Prof
> Wagner e Morgado, se nao foi o marco inicial e corajoso de uma
> revolucao pedagogica, foi, ao menos, o alvorecer de tudo isso. E nos
> somos felizardos por termos em nossa lista estes dois Prof's que
> iniciaram esta revolucao : Wagner e Morgado.
>
> Seria excelente que estes Profs dessem continuidade ao que comecaram
> com tanto brilho e eficiencia !
>
> Para que esta mensagem nao fique totalmente fora de nossa tradicao,
> aqui vai uma joia do Geometria II :
>
> 1)Sejam "a", "b", "c" e "d" os lados de um quadrilatero ciclico.
> Mostre qua a area S desse quadrilatero pode ser expressa como :
> S=Raiz_Quadrada((p-a)(p-b)(p-c)(p-d)) onde p e o semi-perimetro.
>
> 2)Se o quadrilatero e incritivel e circunscritivel, entao :
> S=Raiz_Quadrada(abcd)
>
> PROBLEMA : Se ABC e um triangulo e C o circulo inscrito nele, sejam
> C1, C2 e C3 as tres partes da area do triangulo que nao pertencem ao
> circulo. Calcule cada uma, separadamente, em funcao dos lados do
> triangulo.
>
> Um Abraco a Todos
> Paulo Santa Rita
> 6,,170103
>
>
>
>
>> From: "Leonardo Borges Avelino" <[EMAIL PROTECTED]>
>> To: <[EMAIL PROTECTED]>
>> Subject: [obm-l] Livro Geometria
>> Date: Thu, Jan 16, 2003, 10:07 PM
>>
>>
>> Caros amigos:
>>
>>  Estava na casa de meu amigo e ele me mostrou um livro
>> >impressionante, que
>> se chama: Geometria II dos prof.s Eduardo Wagner, Augusto Morgado e
>> >Miguel.
>> Pergunta:
>>
>> Onde consigo este livro? e (desculpem-me se a pergunta for idiota) se
>> >existe o Geometria I?
>>
>>
>> Valeu!!
>> Leonardo Borges
>
>
>
> _
> MSN Hotmail, o maior webmail do Brasil. http://www.hotmail.com
>
> =
> Instruções para entrar na lista, sair da lista e usar a lista em
> http://www.mat.puc-rio.br/~nicolau/olimp/obm-l.html
> O administrador desta lista é <[EMAIL PROTECTED]>
> =
>
>



=
Instruções para entrar na lista, sair da lista e usar a lista em
http://www.mat.puc-rio.br/~nicolau/olimp/obm-l.html
O administrador desta lista é <[EMAIL PROTECTED]>
=



Re: [obm-l] Re: [obm-l] Dúvida???

2003-01-19 Thread A. C. Morgado



Por exemplo, se n=5, o conjunto eh {1, 2, ...,10}. Considere o subconjunto
{1, 2, 3, 4, 5, 6}. Se x=4, um elemento do subconjunto que não eh multiplo
de x eh o 6. Logo, de acordo com a prova dada 4 e 6 sao primos entre si!
Morgado

Wagner wrote:

  
  
  Oi para todos !
   
  Sim. Aqui vai a prova :
  Suponha que m elementos do subconjunto
sejam  múltiplos de x.
  Para x > 1, temos m < n + 1 . Logo
existe  pelo menos um elemento do subconjunto que não
  é múltiplo de x . Seja y esse elemento
. Como  x =< 2n , então pelo menos x e y
  são primos entre si.
   
  André T.
   
   
  

- Original Message - 

From:
Danilo Artigas


To:
obm-l


Sent: Sunday, January 19, 2003 2:24AM

Subject: [obm-l] Dúvida???


  Por que podemos garantir que em qualquer
   subconjunto com n + 1 elementos do conjunto {1, 2, 3, ..., 2n} existem
   pelo menos dois elementos que são primos entre  si?







Re: [obm-l] probabilidade

2003-01-19 Thread A. C. Morgado
Ha C(n, 3) = n(n-1)(n-2)/6 modos de retirar 3 etiquetas e n-2 modos de 
retirar 3 etiquetas com numeros consecutivos [123, 234,..., 
(n-2)(n-1)n]. A resposta eh o quociente,  6/[n(n-1)]
Morgado
amurpe wrote:

Por favor me ajudem nessa problema.

Numa urna são depositadas n etiquetas numeradas de 1 a 
n .Tres etiquetas são sorteadas ( sem reposição).Qual a 
probabilidade de que os numeros sorteados sejam 
consecutivos?

obrigado.

Amurpe




__
E-mail Premium BOL
Antivírus, anti-spam e até 100 MB de espaço. Assine já!
http://email.bol.com.br/


=
Instruções para entrar na lista, sair da lista e usar a lista em
http://www.mat.puc-rio.br/~nicolau/olimp/obm-l.html
O administrador desta lista é <[EMAIL PROTECTED]>
=




=
Instruções para entrar na lista, sair da lista e usar a lista em
http://www.mat.puc-rio.br/~nicolau/olimp/obm-l.html
O administrador desta lista é <[EMAIL PROTECTED]>
=



Re: [obm-l] Retorno da questão da divisão euclidiana (FGV-SP)

2003-01-19 Thread A. C. Morgado



O sistema 

d - 8D = 24

d + D = 320 
dah, subtraindo as equaçoes, 9D=296 . Ou seja, D nao dah inteiro.
O problema nao tem soluçao!

Provavel erro: Onde estah soma 
do dividendo, do divisor e do resto  deveria ser 
soma do quociente, do dividendo, do divisor e do resto.
Morgado


[EMAIL PROTECTED] wrote:

Olá pessoal,  
  
Olá pessoal, 
  
Estou com dúvidas nesta questão da FVG:  
  
(F.G.V-SP)Numa divisão, o quociente é 8 e o resto, 24. Sabe-se que a soma
do dividendo, do divisor e do resto é 344. Então, a diferença dividendo menos
divisor é:  
  
Resp: 248 
  
Foi dito na lista que eu poderia montar o sistema: 
  
  
d = 8D + 24
  
D + d + 24 = 344 
  
  
d - 8D = 24
  
d + D = 320 
  
Eu concordo, o sistema está correto o problema é que quando eu resolvi este
sistema, conheci os valores de d e D e então  eu fiz d-D que o enunciado
pede e cheguei a uma resposta de 320 e a resposta que o gabarito dá como
certa é 248. 
  
  
   
  
  
  
  
  


Re: [obm-l] Probabilidade (Urna)

2003-01-19 Thread A. C. Morgado
O primeiro ganha se:
i) tira azul   p=1/3
ii) tira vermelha, seu adversario tira vermelha, tira azul p = 
(2/3)(2/3)(1/3)
iii) A   p = (2/3)(2/3)(2/3)(2/3)(1/3)
..
Basta somar. Eh uma pg de razao 4/9 e primeiro termo 1/3.

Gabriel Pérgola wrote:

Boa noite lista,

Gostaria da explicação da resolução deste exercício:

Um jogo para duas pessoas consiste em uma urna com 2 bolas vermelhas e 1
azul. Ganha o jogo quem retirar da urna a bola azul. Caso um jogador retire
uma bola vermelha, essa volta para a urna, e o outro jogador faz sua
retirada. Os jogadores vão alternando suas retiradas até que saia a bola
azul. Todas as bolas têm a mesma probabilidade de serem retiradas. A
probabilidade do primeiro a jogar ganhar o jogo, isto é, em uma de suas
retiradas pegar a bola azul, vale?

A resposta é 3/5. Mas não consegui entender o porquê.


Gabriel


=
Instruções para entrar na lista, sair da lista e usar a lista em
http://www.mat.puc-rio.br/~nicolau/olimp/obm-l.html
O administrador desta lista é <[EMAIL PROTECTED]>
=





=
Instruções para entrar na lista, sair da lista e usar a lista em
http://www.mat.puc-rio.br/~nicolau/olimp/obm-l.html
O administrador desta lista é <[EMAIL PROTECTED]>
=



Re: [obm-l] duvida

2003-01-19 Thread A. C. Morgado
1) Seja p(n) a probabilidade de sheila ganhar a n-esima partida. Para 
que isso aconteça, ou:
i) sheila ganha a n-esima partida e ganha a partida anterior
ou
ii) sheila ganha a n-esima partida e perde a partida anterior
A prob. do primeiro caso vale p(n-1).0,6 e a prob. do segundo caso vale 
[1-p(n-1)].0,4
Entao p(n) =  p(n-1).0,6 + [1-p(n-1)].0,4
p(n) = 0,2 p(n-1) + 0,4
Alem disso p(1) = 0,4
A resposta eh  p(n) = 0,5 - [(0,2^n)/2]



amurpe wrote:

Oi pessoal , meu nome é antonio murpe sou novo na lista. 
tenho 16 anos e gosto de estudar matemática.
estou tentando resolver alguns problemas do livro 
matematica do ensino médio volume : 2 , da coleção do 
professor de matematica , os problemas são muito 
interessantes , mas muito dificeis , gostaria que voces 
me dessem uma ajuda.

1)sheila e helena disputam uma serie de partidas.cada 
partida é iniciada por quem venceu a partida anterior.em 
cada partida , quem a iniciou tem a probabilidade de 0,6 
de ganhá-la e probabilidade 0,4 de perdê-la . Se Helena 
iniciou a primeira partida , qual é a probabilidade de 
Sheila ganhar a n-ésima partida?

2) um circulo foi dividido em n( maior ou igual a 2) 
setores .de quantos modos podemos colori-los , cada 
setor com uma cor , se dispomos de k ( maior que 2) 
cores diferentes e setores adjacentes não devem ter a 
mesma cor?.

Os problemas estão ligados as sequencias recorrentes.

desde já , obrigado.

abraços, Amurpe


__
E-mail Premium BOL
Antivírus, anti-spam e até 100 MB de espaço. Assine já!
http://email.bol.com.br/


=
Instruções para entrar na lista, sair da lista e usar a lista em
http://www.mat.puc-rio.br/~nicolau/olimp/obm-l.html
O administrador desta lista é <[EMAIL PROTECTED]>
=




=
Instruções para entrar na lista, sair da lista e usar a lista em
http://www.mat.puc-rio.br/~nicolau/olimp/obm-l.html
O administrador desta lista é <[EMAIL PROTECTED]>
=



Re: [obm-l] Polinômios

2003-01-19 Thread A. C. Morgado



Uma pergunta que todos estao querendo fazer: que droga de gabarito eh esse
que eh tao cheio de erros?
Quanto ao problema, a soluçao eh:
2m+3n-p=0
m+2n-5p=0
Da primeira equaçao, p=2m+3n. Substituindo na segunda, m+2n-10m-15n=0, ou
seja, -9m = 13n.
m+n+p = -13n/9+n-26n/9 +3n = -n/3. Como n eh arbitrario, a resposta do gabarito
(e a sua tambem) esta errada.
Morgado
[EMAIL PROTECTED] wrote:

Olá pessoal, 
  
Alguém sabe resolver esta questão: 
  
(PUC-MG) Se o polinômio p(x)= (2m + 3n-p)x^2 + (m + 2n - 5p)x é identicamente
nulo, a soma m+n+p é igual a: 
  
Obs: Meu gabarito diz que a resposta é -3, mas eu estou chegando a resposta
de -6 que é uma das alternativas.  
  
  
  


Re: [obm-l] Dúvida: Qual a proposta da lista? Ensino Médio ou Superior?

2003-01-20 Thread A. C. Morgado



Embora eu nao seja o dono da lista, creio que quase todos tem o maior prazer
em ajudar, haja vista que nenhum problema que voce enviou para a lista ficou
sem resposta. O que eu acho, e alguns outros acham tambem, eh que talvez
a quantidade de erros do seu fasciculo esteja mais atrapalhando do que ajudando.
Perguntei qual era o fasciculo para nao permitir que minha filha eventualmente
o leia.
Vou tomar a liberdade de propor uma questao ah lista, em especial aos que
fizeram vestibular recentemente. Que material voces  sugerem ao Rafael em
substituiçao aos que ele estah usando? Eu pensaria nos livros do Iezzi ou
do Manoel Paiva.
Morgado


[EMAIL PROTECTED] wrote:

Olá Morgado e a todos amigos da lista, 
  
Foi até bom o Sr. me perguntar isso...eu estou estudando por uma série de
fascículos contendo todas as disciplinas do ensino médio, inclusive matemática.
Estes fascículos são da nova cultural e foram publicados em bancas de jornais
em 1997 e 1998 mais ou menos.  
Eu gostaria de aproveitar o ensejo e dizer que quando eu vi este grupo de
discussões no site da OBM eu achei fantástico a proposta do Nicolau Saldanha.
Proposta esta que está direcionada à discussão de conteúdos do ensino médio
e 
superior em matemática, mas acho que existem pessoas que não estão entendendo
isto direito. Eu acabei de terminar meu curso de ciências biológicas e quero
obter o máximo de conhecimentos em matemática pelo menos da matemática do
ensino médio, por questões que só cabe a mim. 
Posso afirmar que 95% das questões que coloco aqui são deste meu fascículo,
que salvo os problemas do gabarito, é ótimo. Portanto, um dos motivos é este,
o outro motivo, é claro, se refere a minha dúvida na resolução dos exercícios. 
  
Tenho percebido que existem pessoas maravilhosas aqui na lista que auxiliam
outros na resolução de problemas e tem me auxiliado muito como o Sr. e tantos
outros, mas existem pessoas que mesmo não querendo ajudar atrapalham me enviando
e-mais "off-lista" dizendo que as minhas questão são muito fáceis. Como já
foi supra citado, 95% das questões que coloco aqui  na lista são do meu fascículo
e destas 98% são de vestibulares. Daí, eu gostaria que vcs pensassem comigo...se
o Nicolau criou está lista para discussões de nível de ENSINO MÉDIO e/ou
superior e uma questão de vestibular é mais difícil que um simples exercício
de fixação que encontramos em livros, pois os vestibulares devem manter um
certo nível nas questões para que a relação candidatos e aprovados se mantenha
sempre estável, e sendo 98% das questões que envio serem de vestibular vcs
hão de concordar que quem acha que as minhas questoes são fáceis está na
verdade comparando um conhecimento de ensino superior com matematica de ensino
médio. Deve acontecer duas três situações: 
  
Ou essas pessoas saiam desta lista e vão participar de uma que envolva somente
nivel superior, pois elas declaram que não gostam de responder questões de
vestibular (como as que envio)...nós não aprendemos que os incomodados que
se mudam ? Ainda bem que a maioria aqui, formado em matemática, não acha
ruim questões de nível de Ensino Médio. 
Ou o Nicolau muda a proposta da lista e coloca somente discussões de nível
superior (que eu acho totalmente incabível, pois nas olimpiadas de matematica
há outros niveis diferentes do nível superior e o nível de Ensino médio está
incluso) 
Ou estas pessoas ao invés de ficarem mandando questão para eu parar de mandar
questões parassem de pi(censurado) e deixassem aquelas pessoas
que estão sendo compreensivas e a quem eu sou totalmente grato auxiliarem
não só a mim, mas também a todos que, por ventura, tenha as mesmas dúvidas
que as minhas. 
  
  
  
Resumo: NÃO SÃO AS QUESTÕES QUE SÃO FÁCEIS, MAS SIM O NÍVEL E CONHECIMENTOS
MATEMÁTICOS DE QUEM ESTÁ RECLAMANDO QUE SÃO ALTOS! NÃO ACHAM? POIS AS MINHAS
QUESTÕES SÃO COMPOSTAS DE 98% DE VESTIBULARES, INCLUSIVE O ITA,FUVEST ETC... 
  
  
  
Está aí o meu desabafo para aqueles que estão me enviando e-mails reclamando
da facilidade das questões, por exemplo, do ITA. Que Ironia!! AHAHAHAHAH
:-)  
  
Obs: É claro que para aqueles que já estão a um bom tempo neste mundo da
matemática as questões destes vestibulares, inclusivo, o ITA são fáceis (somente
estas exceções)
  
  
  


Re: [obm-l] análise combinatória

2003-01-21 Thread A. C. Morgado



Ha dois tipos de numeros: os que terminam em 0 e os que terminam em 5.
i) 1 modo de selecionar o ultimo digito (0), 4 de selecionar o primeiro (5,
6, 7 ou 8), 8 de selecionar o segundo (deve ser diferente do primeiro e do
ultimo), 7 o terceiro, 6 o quarto. Ha 1x4x8x7x6 = 1 344 numeros terminados
em 0.
ii) 1 modo de selecionar o ultimo digito (5), 3 de selecionar o primeiro
(6, 7 ou 8), 8 de selecionar o segundo (deve ser diferente do primeiro e
do ultimo), 7 o terceiro, 6 o quarto. Ha 1x3x8x7x6 = 1 008 numeros terminados 
em 5.
A resposta eh 1 344 + 1 008 = 2 352.

[EMAIL PROTECTED] wrote:

Olá, 
  
Como resolver está questão: 
  
O total de números formados com algarismos distintos, maiores que 5 e
menores que 9 e que são divisíveis por 5 é : 
  
Gabarito: 2352  
  
  
  
  
  
  
  
  


Re: [obm-l] análise real.

2003-01-21 Thread A. C. Morgado
Um comentario sobre notaçao:
O conjunto dos reais sempre foi representado por R (podem pegar qualquer 
livro americano ou qualquer frances antigo para conferir). Quando 
começou a tal da Matematica Moderna, franceses e belgas (mais 
precisamente, valoes) começaram a usar um R esquisito (esquisito eh 
bondade minha) para representar o conjunto dos reais. Vamos parar com 
isso e, principalmente nesta lista, vamos parar de tentar reproduzir o R 
esquisito escrevendo IR (fica parecendo que eh o conjunto dos irracionais!)
Alem do mais, essa esquisitice so aparece em livros franceses e 
didaticos paulistas, que copiaram dos franceses.
Vamos usar R para os reais.
Morgado

[EMAIL PROTECTED] wrote:

seja f:IR->IR contínua e lim{f(x)/x,x->0}=L < oo.

prove que f(0)=0

Obrigado.

"Mathematicus nascitur, non fit"
Matemáticos não são feitos, eles nascem
---
Gabriel Haeser
www.gabas.cjb.net


--
Use o melhor sistema de busca da Internet
Radar UOL - http://www.radaruol.com.br



=
Instruções para entrar na lista, sair da lista e usar a lista em
http://www.mat.puc-rio.br/~nicolau/olimp/obm-l.html
O administrador desta lista é <[EMAIL PROTECTED]>
=





=
Instruções para entrar na lista, sair da lista e usar a lista em
http://www.mat.puc-rio.br/~nicolau/olimp/obm-l.html
O administrador desta lista é <[EMAIL PROTECTED]>
=



Re: [obm-l] combinatória

2003-01-23 Thread A. C. Morgado




O primeiro numero do segredo pode ser escolhido de 100 modos, o segundo tambem
de 100 modos...
A resposta eh 100^4 = 10^8.

[EMAIL PROTECTED] wrote:
Olá pessoal, 
 
Vejam a questão:] 
 
(UFCE) Um botão de um cofre tem os números 00, 01, 02, 03...,99. O segredo
dele é uma sequência de 4 números do botão. Assim, 15-11-18-97 ou 11-15-18-97
ou 00-00-43-62 são exemplos de segredos. O número total dos possíveis segredos
é igual a :   
 
Resp: 10^8 
 
Obs: Eu até percebi que pode ser resolvido utilizando o principio fundamental
da contagem, mas ainda não tenho a "manha" de utilizá-lo.





Re: [obm-l] funções

2003-01-23 Thread A. C. Morgado




f(6) = f(5) = 3.

[EMAIL PROTECTED] wrote:
Olá pessoal, 
 
Vejam a questão: 
 
Se f :N--->N é tal que: 
f(n)=n/2, se n for par ou 
f(n)=(n+1)/2, se n for ímpar,  
 
Como provar que existem números distintos p e q tais que f(p)=f(q) ? 
  





Re: [obm-l] duvida

2003-01-23 Thread A. C. Morgado




Ha um problema nessa soluçao. Os setores 1 e n-1 nao tem necessariamente
cores diferentes.
Na verdade, chamando a resposta de P(n), a melhor maneira de determinar P(n)
 eh permitir que os setores 1 e n  possam ter cores iguais, o que daria 
 k * (k-1)^(n-1) 
e descontar o que foi contado indevidamente, ou seja, o numero de maneiras de colorir pondo cores iguais nos setores 1 e n (mas aih tudo se passa como se eles formassem um unico setor), P(n-1).
A recorrencia que permite resolver o problema eh 
P(n) =  k * (k-1)^(n-1) - P(n-1).
Desculpem nao terminar. Estah chegando o tecnico para fazer um upgrade no computador.




amurpe wrote:

  Cluadio , valeu .entendi a sua solução foi bem detalhada 
e me facilitou muito.

Obrigado,

um abraço.

Amurpe. 



  
  
Acho que o segundo problema sai assim:

Numere os setores 1, 2, ..., n de forma que k seja adja

  
  cente a k+1 (1 <= k
  
  
<= n-1) e n seja adjacente a 1.

Inicialmente, temos k escolhas para a cor do setor 1.
Após colorido 1, temos k-

  
  1 escolhas para a cor do setor 2, que tem de ser
  
  
diferente da do setor 1.
Após coloridos 1 e 2, temos k-

  
  1 escolhas para a cor do setor 3, que tem de
  
  
ser diferente da do setor 2.
..
Após coloridos 1, 2, ..., n-2, temos k-

  
  1 escolhas para a cor do setor n-1,
  
  
que tem de ser diferente da do setor n-2.
Finalmente, após coloridos 1, ..., n-1, temos apenas k-

  
  2 escolhas para a cor
  
  
do setor n, uma vez que esta cor tem de ser diferente d

  
  a cor dos setores n-1
  
  
e 1.

Número de maneiras = k * (k-1)^(n-2) * (k-2)

Uma variante interessante é usar setores iguais e consi

  
  derar indistinguíveis
  
  
duas configurações de cores que podem ser obtidas uma d

  
  a outra por meio de
  
  
uma rotação (assim, por exemplo, com 5 setores e 3 core

  
  s, as configurações
  
  
ABACB, BACBA, ACBAB, CBABA e BABAC seriam contadas como

  
   uma só).
  
  
Um abraço,
Claudio.

- Original Message -
From: "amurpe" <[EMAIL PROTECTED]>
To: <[EMAIL PROTECTED]>
Sent: Monday, January 13, 2003 3:25 PM
Subject: [obm-l] duvida


Oi pessoal , meu nome é antonio murpe sou novo na lista

  
  .
  
  
tenho 16 anos e gosto de estudar matemática.
estou tentando resolver alguns problemas do livro
matematica do ensino médio volume : 2 , da coleção do
professor de matematica , os problemas são muito
interessantes , mas muito dificeis , gostaria que voces
me dessem uma ajuda.

1)sheila e helena disputam uma serie de partidas.cada
partida é iniciada por quem venceu a partida anterior.e

  
  m
  
  
cada partida , quem a iniciou tem a probabilidade de 0,

  
  6
  
  
de ganhá-la e probabilidade 0,4 de perdê-la . Se Helena
iniciou a primeira partida , qual é a probabilidade de
Sheila ganhar a n-ésima partida?

2) um circulo foi dividido em n( maior ou igual a 2)
setores .de quantos modos podemos colori-los , cada
setor com uma cor , se dispomos de k ( maior que 2)
cores diferentes e setores adjacentes não devem ter a
mesma cor?.

Os problemas estão ligados as sequencias recorrentes.

desde já , obrigado.

abraços, Amurpe


___

  
  ___
  
  
E-mail Premium BOL
Antivírus, anti-spam e até 100 MB de espaço. Assine já!
http://email.bol.com.br/


===

  
  ==
  
  
Instruções para entrar na lista, sair da lista e usar a

  
   lista em
  
  
http://www.mat.puc-rio.br/~nicolau/olimp/obm-l.html
O administrador desta lista é <[EMAIL PROTECTED]>
===

  
  ==
  
  
===

  
  ==
  
  
Instruções para entrar na lista, sair da lista e usar a

  
   lista em
  
  
http://www.mat.puc-rio.br/~nicolau/olimp/obm-l.html
O administrador desta lista é <[EMAIL PROTECTED]>
===

  
  ==
  
  
 
__
E-mail Premium BOL
Antivírus, anti-spam e até 100 MB de espaço. Assine já!
http://email.bol.com.br/


=
Instruções para entrar na lista, sair da lista e usar a lista em
http://www.mat.puc-rio.br/~nicolau/olimp/obm-l.html
O administrador desta lista é <[EMAIL PROTECTED]>
=


  






Re: [obm-l] probabilidade

2003-01-23 Thread A. C. Morgado
3) A prob. de sair uma criança certa eh 1; depois disso, a prob. de sair 
uma bandeira certa (ouseja, que nao seja a do paihs da criança eh 4/5.
A resposta eh 1 x (4/5) = 4/5.

amurpe wrote:

Pessoal , tentei resolver as questões abaixo de 
vestibulares antigos e senti muita dúvida.

Por favor me deem uma ajuda.

Obrigado e um abraço.

Amurpe.

1)Um adivinho diz ser capaz de ler o pensamento de outra 
pessoa.é feita a seguinte experiencia: seis cartas( 
numeradas de 1 a 6) são dadadas á pessoa, que concentra 
sua atenção em duas delas .O adivinho terá que descobrir 
essas duas cartas.se o adivinho estiver apenas "chutando"
, qual a probabilidade dele acertar as duas cartas , nas 
quais a outra pessoa concentra a atenção?

2)numa gaveta há dez pares de distintos de meias , mas 
ambos pes de um dos pares estão rasgados. Tirando-se da 
gaveta um pé de meia por vez , ao acaso , a 
probabilidade de tirarmos dois pés de meia do mesmo 
par , não rasgados , fazendo  2 retiradas é:...
( resp: 5/20)   


3) em uma sala existem cinco crianças: uma brasileira , 
uma italiana , uma japonesa , uma inglesa e uma 
francesa.Em uma urna existem 5 bandeiras correspondentes 
aos países de origem dessas crianças : Brasil,itália , 
japão , Inglaterra e França.
uma criança  e uma bandeira são selecionadas ao acaso , 
respectivamente, da sala e da urna.A probabilidade de 
que a criança não receba a sua bandeira vale:...( resp; 
20/25)

4) uma urna contem bolas numeradas de 1 a 9 .sorteiam-
se , com reposição , duas bolas.A probabilidade de que o 
numero da segunda bola seja estritamente maior que a 
primeira é :  ( resp: 36/81).





__
E-mail Premium BOL
Antivírus, anti-spam e até 100 MB de espaço. Assine já!
http://email.bol.com.br/


=
Instruções para entrar na lista, sair da lista e usar a lista em
http://www.mat.puc-rio.br/~nicolau/olimp/obm-l.html
O administrador desta lista é <[EMAIL PROTECTED]>
=


 



=
Instruções para entrar na lista, sair da lista e usar a lista em
http://www.mat.puc-rio.br/~nicolau/olimp/obm-l.html
O administrador desta lista é <[EMAIL PROTECTED]>
=



Re: [obm-l] probabilidade

2003-01-23 Thread A. C. Morgado
2) a prob. de o primeiro peh ser  certo eh 18/20 = 9/10. Sepois disso, a 
prob. de o segundo peh ser certo eh 1/19. A resposta eh  (9/10) x (1/19) 
= 9/190.

amurpe wrote:

Pessoal , tentei resolver as questões abaixo de 
vestibulares antigos e senti muita dúvida.

Por favor me deem uma ajuda.

Obrigado e um abraço.

Amurpe.

1)Um adivinho diz ser capaz de ler o pensamento de outra 
pessoa.é feita a seguinte experiencia: seis cartas( 
numeradas de 1 a 6) são dadadas á pessoa, que concentra 
sua atenção em duas delas .O adivinho terá que descobrir 
essas duas cartas.se o adivinho estiver apenas "chutando"
, qual a probabilidade dele acertar as duas cartas , nas 
quais a outra pessoa concentra a atenção?

2)numa gaveta há dez pares de distintos de meias , mas 
ambos pes de um dos pares estão rasgados. Tirando-se da 
gaveta um pé de meia por vez , ao acaso , a 
probabilidade de tirarmos dois pés de meia do mesmo 
par , não rasgados , fazendo  2 retiradas é:...
( resp: 5/20)   


3) em uma sala existem cinco crianças: uma brasileira , 
uma italiana , uma japonesa , uma inglesa e uma 
francesa.Em uma urna existem 5 bandeiras correspondentes 
aos países de origem dessas crianças : Brasil,itália , 
japão , Inglaterra e França.
uma criança  e uma bandeira são selecionadas ao acaso , 
respectivamente, da sala e da urna.A probabilidade de 
que a criança não receba a sua bandeira vale:...( resp; 
20/25)

4) uma urna contem bolas numeradas de 1 a 9 .sorteiam-
se , com reposição , duas bolas.A probabilidade de que o 
numero da segunda bola seja estritamente maior que a 
primeira é :  ( resp: 36/81).





__
E-mail Premium BOL
Antivírus, anti-spam e até 100 MB de espaço. Assine já!
http://email.bol.com.br/


=
Instruções para entrar na lista, sair da lista e usar a lista em
http://www.mat.puc-rio.br/~nicolau/olimp/obm-l.html
O administrador desta lista é <[EMAIL PROTECTED]>
=


 



=
Instruções para entrar na lista, sair da lista e usar a lista em
http://www.mat.puc-rio.br/~nicolau/olimp/obm-l.html
O administrador desta lista é <[EMAIL PROTECTED]>
=



Re: [obm-l] probabilidade

2003-01-23 Thread A. C. Morgado
4) A prob. de serem iguais eh 1 x 1/9 (o primeiro pode ser qualquer e o 
segundo deve ser igual ao primeiro). A prob. de serem diferentes eh 1 - 
(1/9) = 8/9. Logo, a prob. de o segundo ser maior que o primeiro eh 4/9 
e a de o segundo ser menor que o primeiro eh 4/9.

amurpe wrote:

Pessoal , tentei resolver as questões abaixo de 
vestibulares antigos e senti muita dúvida.

Por favor me deem uma ajuda.

Obrigado e um abraço.

Amurpe.

1)Um adivinho diz ser capaz de ler o pensamento de outra 
pessoa.é feita a seguinte experiencia: seis cartas( 
numeradas de 1 a 6) são dadadas á pessoa, que concentra 
sua atenção em duas delas .O adivinho terá que descobrir 
essas duas cartas.se o adivinho estiver apenas "chutando"
, qual a probabilidade dele acertar as duas cartas , nas 
quais a outra pessoa concentra a atenção?

2)numa gaveta há dez pares de distintos de meias , mas 
ambos pes de um dos pares estão rasgados. Tirando-se da 
gaveta um pé de meia por vez , ao acaso , a 
probabilidade de tirarmos dois pés de meia do mesmo 
par , não rasgados , fazendo  2 retiradas é:...
( resp: 5/20)   


3) em uma sala existem cinco crianças: uma brasileira , 
uma italiana , uma japonesa , uma inglesa e uma 
francesa.Em uma urna existem 5 bandeiras correspondentes 
aos países de origem dessas crianças : Brasil,itália , 
japão , Inglaterra e França.
uma criança  e uma bandeira são selecionadas ao acaso , 
respectivamente, da sala e da urna.A probabilidade de 
que a criança não receba a sua bandeira vale:...( resp; 
20/25)

4) uma urna contem bolas numeradas de 1 a 9 .sorteiam-
se , com reposição , duas bolas.A probabilidade de que o 
numero da segunda bola seja estritamente maior que a 
primeira é :  ( resp: 36/81).





__
E-mail Premium BOL
Antivírus, anti-spam e até 100 MB de espaço. Assine já!
http://email.bol.com.br/


=
Instruções para entrar na lista, sair da lista e usar a lista em
http://www.mat.puc-rio.br/~nicolau/olimp/obm-l.html
O administrador desta lista é <[EMAIL PROTECTED]>
=


 



=
Instruções para entrar na lista, sair da lista e usar a lista em
http://www.mat.puc-rio.br/~nicolau/olimp/obm-l.html
O administrador desta lista é <[EMAIL PROTECTED]>
=



Re: [obm-l] probabilidade

2003-01-23 Thread A. C. Morgado
1) O adivinho tem C(6,2)=15 chutes possiveis e apenas um correto. A 
resposta eh  1/15.

amurpe wrote:

Pessoal , tentei resolver as questões abaixo de 
vestibulares antigos e senti muita dúvida.

Por favor me deem uma ajuda.

Obrigado e um abraço.

Amurpe.

1)Um adivinho diz ser capaz de ler o pensamento de outra 
pessoa.é feita a seguinte experiencia: seis cartas( 
numeradas de 1 a 6) são dadadas á pessoa, que concentra 
sua atenção em duas delas .O adivinho terá que descobrir 
essas duas cartas.se o adivinho estiver apenas "chutando"
, qual a probabilidade dele acertar as duas cartas , nas 
quais a outra pessoa concentra a atenção?

2)numa gaveta há dez pares de distintos de meias , mas 
ambos pes de um dos pares estão rasgados. Tirando-se da 
gaveta um pé de meia por vez , ao acaso , a 
probabilidade de tirarmos dois pés de meia do mesmo 
par , não rasgados , fazendo  2 retiradas é:...
( resp: 5/20)   


3) em uma sala existem cinco crianças: uma brasileira , 
uma italiana , uma japonesa , uma inglesa e uma 
francesa.Em uma urna existem 5 bandeiras correspondentes 
aos países de origem dessas crianças : Brasil,itália , 
japão , Inglaterra e França.
uma criança  e uma bandeira são selecionadas ao acaso , 
respectivamente, da sala e da urna.A probabilidade de 
que a criança não receba a sua bandeira vale:...( resp; 
20/25)

4) uma urna contem bolas numeradas de 1 a 9 .sorteiam-
se , com reposição , duas bolas.A probabilidade de que o 
numero da segunda bola seja estritamente maior que a 
primeira é :  ( resp: 36/81).





__
E-mail Premium BOL
Antivírus, anti-spam e até 100 MB de espaço. Assine já!
http://email.bol.com.br/


=
Instruções para entrar na lista, sair da lista e usar a lista em
http://www.mat.puc-rio.br/~nicolau/olimp/obm-l.html
O administrador desta lista é <[EMAIL PROTECTED]>
=


 



=
Instruções para entrar na lista, sair da lista e usar a lista em
http://www.mat.puc-rio.br/~nicolau/olimp/obm-l.html
O administrador desta lista é <[EMAIL PROTECTED]>
=



Re: [obm-l] combinatória

2003-01-23 Thread A. C. Morgado




Rafael:
esse problema caiu na UERJ, a resposta eh 48. Mas a tabela nao era 3 por3
e sim 2 por 3  (2 linhas e 3 colunas).

Rafael wrote:

  Olá Pessoal!

Resolvendo uma questão que recebi encontrei uma
resposta muito diferente das alternativas. Disseram-me
que a resposta era alternativa d) 48. Porém ao
resolver o problema eu encontrei como resposta 3348,
maneiras!!

Se alguém puder tentar fazer pra ver se eu pensei
alguma coisa errada agradeço. Vejam a questão:

  
  
26 - De quantos modos se pode colocar na tabela
abaixo duas letras A, duas letras B e duas letras C,
uma em cada casa, de modo que não haja duas letras
iguais na mesma coluna?
 _ _ _
|_|_|_|
|_|_|_|
|_|_|_|

a) 12
b) 24
c) 36
d) 48
e) 64

  
  
Abraços,

Rafael.

__
Do you Yahoo!?
Yahoo! Mail Plus - Powerful. Affordable. Sign up now.
http://mailplus.yahoo.com
=
Instruções para entrar na lista, sair da lista e usar a lista em
http://www.mat.puc-rio.br/~nicolau/olimp/obm-l.html
O administrador desta lista é <[EMAIL PROTECTED]>
=


  






Re: [obm-l] Sequências

2003-01-27 Thread A. C. Morgado




[(n^2+n)/2]^2 - [(n^2-n)/2]^2 = n^3
É fácil ver que como n e nê têm a mesma paridade, os números entre colchetes
são inteiros.

Wagner wrote:
  
  
 
  
 

  Provar que todo cubo de um número inteiro
é a  diferença de dois quadrados de números inteiros
 
   
 
  André T.






[obm-l] Re: [obm-l] Re: [obm-l] Um tabuleiro de xadrez diferente! Combinatória

2003-01-27 Thread A. C. Morgado




Uma observação:
É impressionante o prazer que os autores dessas questões de vestibular sentem
em enrolar desnecessariamente os enunciados.Custava alguma coisa ter dito
no enunciado 4 peças diferentes?
É claro que a solução do Cláudio está correta e se refere a 4 peças diferentes.
Proponho então um outro problema: e se as peças fossem iguais?
Morgado

Cláudio (Prática) wrote:
  
  
 
  
 

  Uma forma de resolver o problema é através
do  preenchimento de uma linha de cada vez:
 
   
 
  Colocação da primeira peça na primeira
 linha:
 
  - Escolha da primeira peça: 4 (existem
inicialmente  4 peças disponíveis)
 
  - Escolha da coluna: 4 (todas as colunas
estão  disponíveis)
 
  Colocação da segunda peça na segunda  linha:
 
   
  - Escolha
da segunda peça: 3 (uma das peças já  foi utilizada)
 
  - Escolha
da coluna: 3 (a coluna da primeira peça  deve ser evitada)
 
   
  Colocação
da terceira peça na terceira  linha:
 
   
  - Escolha da terceira peça: 2
 
  - Escolha da coluna: 2 
 
   
  Colocação da última peça na última  linha:
 
   
  - Escolha da última peça: 1
 
  - Escolha da  coluna: 1 
  
  
  
  
 
   
 
  Total = 4*4*3*3*2*2*1*1 = 576.
  
 
   
  
- Original Message
- 
   
From:[EMAIL PROTECTED]
   
   
To: [EMAIL PROTECTED]

   
Sent: Monday, January
27, 2003 3:25AM
   
Subject: [obm-l]
Um tabuleiro de xadrezdiferente! Combinatória
   


Olá pessoal,

Como resolver esta questão: 

(FGV-SP) Um tabuleiro especial dexadrez possui 16 casas, dispostas em
4 linhas e 4 colunas. Um jogador desejacolocar 4 peças no tabuleiro,
de tal forma que, em cada linha e cada coluna,seja colocada apenas uma
peça. De quantas maneiras as peças poderão sercolocadas? 

Resp:576 

Obs: Eu pensei no seguinte: 
A únicamaneira de termos em cada linha e cada coluna apenas uma peça
é se estas peçasforem colocadas na diagonal. E como um quadrado possui
2 diagonais a respostaseria o total de maneiras de colocar as peças em
uma diagoanl multiplicado por2. Como eu disponho de 4 peças para colocar
em 4 casas (cada diagonal possui 4casas) eu tenho a seguinte situação:

Em qualquer casa eu tenho 4 escolhas(4 peças) então temos 4^4 =256 agora
multiplicando por 2, pois a outradiagonal também poderia ser temos 256x2=512
    






Re: [obm-l] análise de sinais (funções)

2003-02-05 Thread A. C. Morgado




Voce somou fraçoes e se esqueceu de escrever o denominador da soma.

[EMAIL PROTECTED] wrote:
(FUVEST) Resolva 2x - 3 + 5*[(1/x)
+ 1] <=1 
 
resp:{x e R| x<0} 
 
Obs: Eu tentei resolver mas não cheguei neste resultado, vejam minha resolução
e me digam onde errei: 
2x-3+(5/x)+5<=1 
2x-3+(5/x)+5-1<=0 
2x^2 -3x + 5 + 4x <=0 (Nesta etapa eu multipliquei por x) 
2x^2 + x + 5<=0 
A partir disso percebe-se que delta é igual -39, portanto não há raízes reais
e a resposta não pode ser :{x e R| x<0}. 
 
  





Re: [obm-l] trigonometria (transformação de arcos)

2003-02-05 Thread A. C. Morgado




Voce esta certo. Seu gabarito, como sempre, errado.

[EMAIL PROTECTED] wrote:
(UECE) Se  P= [(sen 40º)/(sen 20º)]
- [(cos 40º)/(cos 20º)], então p^2 - 1 é igual: 
Resp: cotg^2 (20º) 
 
Obs: Será que o resultado não é tg^2 (20º)? 
 
ICQ: 337140512





Re: [obm-l] geometria espacial

2003-02-05 Thread A. C. Morgado




No gomo ha dois "lados" que nao pertencem a esfera.

[EMAIL PROTECTED] wrote:
Oá pessoal, 
 
Uma laranja pode ser considerada uma esfera de raio R, composta de 12 gomos
exatamente "iguais". A área da superfície total de cada gomo é dada por: 
 
resp: (4*pi*R^2)/3 
Obs: A resposta não seria  (pi*R^2)/3 ? Pois se há 12 gomos então a área
superficial de casa gomo é igual a (área superficial total da esfera)/(12).
Será que o gabarito está errado novamente? 
  





Re: [obm-l] Livro Geometria

2003-02-05 Thread A. C. Morgado
Um quadrilatero inscritivel. Um quadrilatero tal que existe uma 
circunferencia que contem os 4 vertices.

Frederico Reis Marques de Brito wrote:



Sobre os livros Geometria I e II e Álgebra I , tentei em vão falar no 
fone indicado pelo Morgado ( até rimou... ). Mas o telefone está 
programado para não receber ligações. Gostaria de saber os tópicos 
abordados nesses livros, os preços e se há alggum site ou algum 
representante da editora em Belo Horizonte. Aproveito para esclarecer 
uma dúvida conceitual. O que vem a ser um quadrilátero cíclico?
Desde já deixo aqui meus agradecimentos.
Frederico.




From: "A. C. Morgado" <[EMAIL PROTECTED]>
Reply-To: [EMAIL PROTECTED]
To: [EMAIL PROTECTED]
Subject: Re: [obm-l] Livro Geometria
Date: Fri, 17 Jan 2003 15:05:29 -0200

Agradecemos as referencias elogiosas do Paulo.
Corrijo as declaraçoes do Wagner:
1) o Geometria 1 eh um bom livro. Claro que, por tratar de conceitos
mais basicos e se dirigir basicamente a um leitor que seja um bom aluno
de SETIMA SERIE, sempre os autores (e leitores mais adiantados) acharao
que se poderia dar um tratamento "axiomaticamente" mais rigoroso; mas o
objetivo era fazer um livro de Geometria para vestibulares dificeis e
concursos de admissao a escolas militares, extremamente procuradas na
epoca; ainda hoje eh um bom livro para IME, ITA, Colegio Naval, etc.
2) Geometria 1, Geometria 2 e Algebra 1 podem ser pedidos a
FC&Z Livros
Rua Carneiro Ribeiro 22 loja A
21050-570  Maria da GraçaRio de JaneiroRJ
Telefax  (21) 2581-2873
Morgado

Paulo Santa Rita wrote:

> Ola Leonardo e demais
> colegas desta lista ... OBM-L,
>
> Foi esse livro que despertou meu interesse pela Matematica.
>
> Voce teve a mesma impressao que eu tive, quando o li pela primeira
> vez, apos adquiri-lo em um sebo ( sebo = livraria de livros usados ).
> Muitas vezes me perguntei o que o tornava tao interessante e diferente
> dos outros ...
>
> Me parece que e porque os autores ( Eduardo Wagner e Augusto Morgado )
> nao perdem tempo provando coisas simples e evidentes, partindo
> imediatamente a exposicao de fatoss espetaculares e inusitadas, que
> dificilmente imaginariamos que ocorrem.
>
> Quando um matematico explica um fenomeno inusitado ( por exemplo : o
> circulo de nove pontos ) ele aguca nossa inteligencia e nos vemos
> claramente que a Matemaica nao se resume a provas de fatos evidentes
> e, portanto, desmotivadoras ( por exemplo : prove que 1+1=2 ).
>
> Um Matematico adulto pode apreciar o formalismo, mas uma mente nova
> geralmente nao aprecia estas coisas ... Prove a uma crianca algo que
> ela duvida e nao suspeia e voce vai conquistar o interesse dela ... me
> parece que e esse simples detalhe que diferencia as grandes obras da
> mesmice e mediocridade que campeia na imensa maioria das obras
> didaticas da matematica ...
>
> As Olimpiadas de Matematica, que todos nos gostamos e admiramos e
> pelas quais fazemos verdadeiros sacrificios e uma continuacao de tudo
> isso ... Por que elas fazem sucesso ? Simplesmente porque as pessoas
> inteligentes odeiam coisas rotineiras e burocraticas, que sao os
> aspectos tipicos do ensino comum ...
>
> Neste sentido, o Livro a que voce se refere, o Geometria II dos Prof
> Wagner e Morgado, se nao foi o marco inicial e corajoso de uma
> revolucao pedagogica, foi, ao menos, o alvorecer de tudo isso. E nos
> somos felizardos por termos em nossa lista estes dois Prof's que
> iniciaram esta revolucao : Wagner e Morgado.
>
> Seria excelente que estes Profs dessem continuidade ao que comecaram
> com tanto brilho e eficiencia !
>
> Para que esta mensagem nao fique totalmente fora de nossa tradicao,
> aqui vai uma joia do Geometria II :
>
> 1)Sejam "a", "b", "c" e "d" os lados de um quadrilatero ciclico.
> Mostre qua a area S desse quadrilatero pode ser expressa como :
> S=Raiz_Quadrada((p-a)(p-b)(p-c)(p-d)) onde p e o semi-perimetro.
>
> 2)Se o quadrilatero e incritivel e circunscritivel, entao :
> S=Raiz_Quadrada(abcd)
>
> PROBLEMA : Se ABC e um triangulo e C o circulo inscrito nele, sejam
> C1, C2 e C3 as tres partes da area do triangulo que nao pertencem ao
> circulo. Calcule cada uma, separadamente, em funcao dos lados do
> triangulo.
>
> Um Abraco a Todos
> Paulo Santa Rita
> 6,,170103
>
>
>
>
>> From: "Leonardo Borges Avelino" <[EMAIL PROTECTED]>
>> To: <[EMAIL PROTECTED]>
>> Subject: [obm-l] Livro Geometria
>> Date: Thu, Jan 16, 2003, 10:07 PM
>>
>>
>> Caros amigos:
>>
>>  Estava na casa de meu amigo e ele me mostrou um livro
>> >impressionante, que
>> se chama: Geometria II dos prof.s Eduardo Wagner, Augusto Morgado e
>> >Miguel.
>> Pergunt

Re: [obm-l] Livro Geometria

2003-02-05 Thread A. C. Morgado
Houve um acidente com um caminhao que destruiu parte da rede telefonica 
(ah!, a Telemar!) da rua da livraria. Escreva uma carta ou aguarde uns dias.
Morgado


Frederico Reis Marques de Brito wrote:



Sobre os livros Geometria I e II e Álgebra I , tentei em vão falar no 
fone indicado pelo Morgado ( até rimou... ). Mas o telefone está 
programado para não receber ligações. Gostaria de saber os tópicos 
abordados nesses livros, os preços e se há alggum site ou algum 
representante da editora em Belo Horizonte. Aproveito para esclarecer 
uma dúvida conceitual. O que vem a ser um quadrilátero cíclico?
Desde já deixo aqui meus agradecimentos.
Frederico.




From: "A. C. Morgado" <[EMAIL PROTECTED]>
Reply-To: [EMAIL PROTECTED]
To: [EMAIL PROTECTED]
Subject: Re: [obm-l] Livro Geometria
Date: Fri, 17 Jan 2003 15:05:29 -0200

Agradecemos as referencias elogiosas do Paulo.
Corrijo as declaraçoes do Wagner:
1) o Geometria 1 eh um bom livro. Claro que, por tratar de conceitos
mais basicos e se dirigir basicamente a um leitor que seja um bom aluno
de SETIMA SERIE, sempre os autores (e leitores mais adiantados) acharao
que se poderia dar um tratamento "axiomaticamente" mais rigoroso; mas o
objetivo era fazer um livro de Geometria para vestibulares dificeis e
concursos de admissao a escolas militares, extremamente procuradas na
epoca; ainda hoje eh um bom livro para IME, ITA, Colegio Naval, etc.
2) Geometria 1, Geometria 2 e Algebra 1 podem ser pedidos a
FC&Z Livros
Rua Carneiro Ribeiro 22 loja A
21050-570  Maria da GraçaRio de JaneiroRJ
Telefax  (21) 2581-2873
Morgado

Paulo Santa Rita wrote:

> Ola Leonardo e demais
> colegas desta lista ... OBM-L,
>
> Foi esse livro que despertou meu interesse pela Matematica.
>
> Voce teve a mesma impressao que eu tive, quando o li pela primeira
> vez, apos adquiri-lo em um sebo ( sebo = livraria de livros usados ).
> Muitas vezes me perguntei o que o tornava tao interessante e diferente
> dos outros ...
>
> Me parece que e porque os autores ( Eduardo Wagner e Augusto Morgado )
> nao perdem tempo provando coisas simples e evidentes, partindo
> imediatamente a exposicao de fatoss espetaculares e inusitadas, que
> dificilmente imaginariamos que ocorrem.
>
> Quando um matematico explica um fenomeno inusitado ( por exemplo : o
> circulo de nove pontos ) ele aguca nossa inteligencia e nos vemos
> claramente que a Matemaica nao se resume a provas de fatos evidentes
> e, portanto, desmotivadoras ( por exemplo : prove que 1+1=2 ).
>
> Um Matematico adulto pode apreciar o formalismo, mas uma mente nova
> geralmente nao aprecia estas coisas ... Prove a uma crianca algo que
> ela duvida e nao suspeia e voce vai conquistar o interesse dela ... me
> parece que e esse simples detalhe que diferencia as grandes obras da
> mesmice e mediocridade que campeia na imensa maioria das obras
> didaticas da matematica ...
>
> As Olimpiadas de Matematica, que todos nos gostamos e admiramos e
> pelas quais fazemos verdadeiros sacrificios e uma continuacao de tudo
> isso ... Por que elas fazem sucesso ? Simplesmente porque as pessoas
> inteligentes odeiam coisas rotineiras e burocraticas, que sao os
> aspectos tipicos do ensino comum ...
>
> Neste sentido, o Livro a que voce se refere, o Geometria II dos Prof
> Wagner e Morgado, se nao foi o marco inicial e corajoso de uma
> revolucao pedagogica, foi, ao menos, o alvorecer de tudo isso. E nos
> somos felizardos por termos em nossa lista estes dois Prof's que
> iniciaram esta revolucao : Wagner e Morgado.
>
> Seria excelente que estes Profs dessem continuidade ao que comecaram
> com tanto brilho e eficiencia !
>
> Para que esta mensagem nao fique totalmente fora de nossa tradicao,
> aqui vai uma joia do Geometria II :
>
> 1)Sejam "a", "b", "c" e "d" os lados de um quadrilatero ciclico.
> Mostre qua a area S desse quadrilatero pode ser expressa como :
> S=Raiz_Quadrada((p-a)(p-b)(p-c)(p-d)) onde p e o semi-perimetro.
>
> 2)Se o quadrilatero e incritivel e circunscritivel, entao :
> S=Raiz_Quadrada(abcd)
>
> PROBLEMA : Se ABC e um triangulo e C o circulo inscrito nele, sejam
> C1, C2 e C3 as tres partes da area do triangulo que nao pertencem ao
> circulo. Calcule cada uma, separadamente, em funcao dos lados do
> triangulo.
>
> Um Abraco a Todos
> Paulo Santa Rita
> 6,,170103
>
>
>
>
>> From: "Leonardo Borges Avelino" <[EMAIL PROTECTED]>
>> To: <[EMAIL PROTECTED]>
>> Subject: [obm-l] Livro Geometria
>> Date: Thu, Jan 16, 2003, 10:07 PM
>>
>>
>> Caros amigos:
>>
>>  Estava na casa de meu amigo e ele me mostrou um livro
>> >impressionante, que
>> se chama: G

Re: [obm-l] Máximos e Mínimos SEM DERIVADAS

2003-02-05 Thread A. C. Morgado
Resolva a equaçao ao contrario. Dah
5x^2 +(21-E)x +16 =0
10x= E-21 (+_) sqrt [(21-E)^2 - 320]
Portanto, (21-E)^2 - 320 deve ser maior ou igual 0.
Daih, E (menor ou igual) 21-sqrt320   ou  E (maior ou igual) 21 +sqrt 320
Eh facil ver ( se x positivo, E>21; se x<0, E<21)  que os primeiros 
valores ocorrem para x negativo e os segundos, para x positivo.
A resposta eh  21 + sqrt320.

Thyago Alexandre Kufner wrote:

Olá colegas da lista

Recebi o seguinte exercício de um aluno:

"Sendo x um nº positivo determine o menor valor de E= 5x + 16/x + 21"

Normal, um exercício simples. Deriva, iguala a zero ...

Mas o que quero propor para a lista é o seguinte: tem como chegar ao
resultado SEM UTILIZAR CÁLCULO?

Proponho esta discussão por causa do seguinte artigo:

http://mathcircle.berkeley.edu/BMC4/Handouts/MaxMin.pdf

Aguardo resposta

Atenciosamente
Prof. Thyago
WebMaster cursinho.hpg.com.br

=
Instruções para entrar na lista, sair da lista e usar a lista em
http://www.mat.puc-rio.br/~nicolau/olimp/obm-l.html
O administrador desta lista é <[EMAIL PROTECTED]>
=


 



=
Instruções para entrar na lista, sair da lista e usar a lista em
http://www.mat.puc-rio.br/~nicolau/olimp/obm-l.html
O administrador desta lista é <[EMAIL PROTECTED]>
=



[Fwd: Re: [obm-l] IME]

2003-02-06 Thread A. C. Morgado




 Estou reenviando esta mensagem pois alguem novo na lista pediu.

 Original Message 

  

  Subject: 
  Re: [obm-l] IME


  Date: 
  Sat, 07 Dec 2002 23:48:36 -0200


  From: 
  "A. C. Morgado" <[EMAIL PROTECTED]>


  To: 
  [EMAIL PROTECTED]


  References: 
  <001201c29e49$a7046700$5225f3c8@wander>

  

 

   1) (cosx)^n = 1 + (senx)^n
 Se n eh par, o segundo membro eh maior ou igual a 1 e a igualdade so sera 
possivel se senx=0, o que da as soluçoes x=k(pi)
 Se n=1, a equaçao eh  cosx - senx = 1
 Multiplique tudo por (sqrt2) / 2 e obtera  cos [x+(pi/4)] = cos(pi/4), que 
da  x=2kpi e tambem a soluçao h = 2kpi- (pi/2)
 Se n eh impar maior que 1, faça x = - z. A equaçao se transforma em  (cosz)^n 
+ (senz)^n = 1. A firmo que essa equaçao so possui as soluçoes  cosz=1 e senz=1.
Basta observar que  (cosz)^2 + (senz)^2 = 1 e que se cosz e senz estiverem
no aberto (0, 1), (cosz)^n + (senz)^n sera estritamente menor que (cosz)^2
+ (senz)^2 = 1. (Eh claro que soluçoes com cosz ou senz negativos, nem pensar).
Entao, as unicas soluçoes serao  z= 2kpi  e  z= 2kpi + (pi/2). Como x = -z,

 
 
Alguem escreveu
 
 
  
  
   
 
   1)
 Encontre todas as soluções reais da equação apresentada abaixo, onde n é
um  número natural.
   
  
cosnx – sennx = 1 
 
   
   
   
   
  
   
   
   




[obm-l] Re: [obm-l] Re: [obm-l] Livros que desenvolvam raciocínio espacial

2003-02-06 Thread A. C. Morgado




Discordo totalmente.
Sugiro os livros do Raymond Smullian (é com i ou é com y?) e o É divertido
resolver problemas, de Luis Lopes e Josimar Silva.
O autor que voce recomenda apareceu no programa do Jô Soares ensinando macetes,
muitos dos quais errados,  para fazer contas mais depressa, talvez ignorando
que calculadoras ja foram inventadas e certamente pensando que Matemática
é uma coleção de macetes para fazer contas mais depressa e encontra-se, no
momento preso, em Brasília.
Morgado
Daniel wrote:

  Não sei se os professores e alunos da lista concordam, mas dois
livros que me ajudaram bastante com matemática olímpica (problemas de
raciocínio) são do Prof. Jonofon Serates (não sei se está correto), são dois
volumes, chama-se "Raciocínio Lógico".

Daniel
- Original Message -
From: "fabio fortes" <[EMAIL PROTECTED]>
To: <[EMAIL PROTECTED]>
Sent: Wednesday, February 05, 2003 8:17 PM
Subject: [obm-l] Livros que desenvolvam raciocínio espacial


  
  
Sei que boa parte é talento, mas gostaria de saber se
alguém conhece livros que trabalhem raciocínio
espacial. Não somente geometria espacial, mas também
questões que usualmente caem em testes de QI e de
lógica.
Obrigado

__
Do you Yahoo!?
Yahoo! Mail Plus - Powerful. Affordable. Sign up now.
http://mailplus.yahoo.com
=
Instruções para entrar na lista, sair da lista e usar a lista em
http://www.mat.puc-rio.br/~nicolau/olimp/obm-l.html
O administrador desta lista é <[EMAIL PROTECTED]>
=

  
  
=
Instruções para entrar na lista, sair da lista e usar a lista em
http://www.mat.puc-rio.br/~nicolau/olimp/obm-l.html
O administrador desta lista é <[EMAIL PROTECTED]>
=


  






Re: [obm-l] inequação

2003-02-06 Thread A. C. Morgado




Em maiusculas o meu comentario! 

[EMAIL PROTECTED] wrote:
Olá pessoal, 
 
Vejam a questão: 
 
(MACK) Resolver a inequação: t + (1/t) <= -2 
 
resp: t e R | t < 0. 
 
Obs: Vejam minha resolução: 
 
t + (1/t) + 2 <= 0 
(t^2 + 2t + 1)/t <= 0 (t # 0) 
Calculando delta chegaremos a delta = 0 
Logo, a equação terá uma raiz (que será -1) e esta terá multiplicidade 2. 
  
Como a equação pede f(t) (vamos chamar assim) <=0 temos que somente t=
-1 satisfaz, pois qualquer valor t pertencente aos reais f(x) ( NAO EH F
QUE SERAH POSITIVA; EH O NUMERADOR QUE SERAH) será positiva, pois delta=
0. Não estou certo? 
 
ICQ: 337140512 
 
  





[obm-l] Re: [obm-l] Re: [obm-l] Re: [obm-l] Livros que desenvolvam raciocínio espacial

2003-02-06 Thread A. C. Morgado




Como professor estou profundamente indignado com p tratamento de professor
dado a tal figura!
Morgado

A. C. Morgado wrote:

  
   Discordo totalmente.
 Sugiro os livros do Raymond Smullian (é com i ou é com y?) e o É divertido 
resolver problemas, de Luis Lopes e Josimar Silva.
 O autor que voce recomenda apareceu no programa do Jô Soares ensinando macetes, 
muitos dos quais errados,  para fazer contas mais depressa, talvez ignorando 
que calculadoras ja foram inventadas e certamente pensando que Matemática 
é uma coleção de macetes para fazer contas mais depressa e encontra-se, no 
momento preso, em Brasília.
 Morgado
 Daniel wrote:
 
 
Não sei se os professores e alunos da lista concordam, mas dois
livros que me ajudaram bastante com matemática olímpica (problemas de
raciocínio) são do Prof. Jonofon Serates (não sei se está correto), são dois
volumes, chama-se "Raciocínio Lógico".

Daniel
- Original Message -
From: "fabio fortes" <[EMAIL PROTECTED]>
To: <[EMAIL PROTECTED]>
Sent: Wednesday, February 05, 2003 8:17 PM
Subject: [obm-l] Livros que desenvolvam raciocínio espacial


  
   
 
  Sei que boa parte é talento, mas gostaria de saber se
alguém conhece livros que trabalhem raciocínio
espacial. Não somente geometria espacial, mas também
questões que usualmente caem em testes de QI e de
lógica.
Obrigado

__
Do you Yahoo!?
Yahoo! Mail Plus - Powerful. Affordable. Sign up now.
http://mailplus.yahoo.com
=
Instruções para entrar na lista, sair da lista e usar a lista em
http://www.mat.puc-rio.br/~nicolau/olimp/obm-l.html
O administrador desta lista é <[EMAIL PROTECTED]>
=

   
   

=
Instruções para entrar na lista, sair da lista e usar a lista em
http://www.mat.puc-rio.br/~nicolau/olimp/obm-l.html
O administrador desta lista é <[EMAIL PROTECTED]>
=


  
 
 
 





Re: [obm-l] Re: [obm-l] Máximos_e_Mínimos_SEM_DERIVADAS

2003-02-06 Thread A. C. Morgado




Oh gente, o problema era arranjar uma soluçao que servisse para um aluno
de oitava serie, nao?
Eh claro que se vale tudo exceto calculo, a soluçao do Salvador eh otima!
Morgado

Salvador Addas Zanata wrote:

  Pode ser assim tambem:

E=5x+16/x+21 >= 2*sqrt(80)+21, usando a desigualdade das medias.


On Wed, 5 Feb 2003, Helder Suzuki wrote:

  
  
 --- Thyago Alexandre Kufner <[EMAIL PROTECTED]> escreveu: >
Olá colegas da lista


  Recebi o seguinte exercício de um aluno:

"Sendo x um nº positivo determine o menor valor de
E= 5x + 16/x + 21"

Normal, um exercício simples. Deriva, iguala a zero
...

Mas o que quero propor para a lista é o seguinte:
tem como chegar ao
resultado SEM UTILIZAR CÁLCULO?

Proponho esta discussão por causa do seguinte
artigo:


  

http://mathcircle.berkeley.edu/BMC4/Handouts/MaxMin.pdf


  Aguardo resposta

Atenciosamente
Prof. Thyago
WebMaster cursinho.hpg.com.br
  

vejamos
y = 5x + 16/x + 21

multiplicando tudo por x, temos que
xy = 5x^2 + 21x + 16
=>
5x^2 + (21-y)x + 16 = 0

Como X é real, o delta não pode ser menor que zero.
portanto:

Delta = (y-21)^2 - 16*5 >= 0
y^2 - 42y + 441 - 16*4 >= 0
y^2 - 42y + 347 >= 0

se voce resolver essa inequação vc encontrará os
intervalos em que não há raiz de números negativos: os
invevalos em que y existe.
(você vai encontrar algo como y >= ... e y <= ..., daí
fica fácil ver o máximo e mínimo locais)

___
Busca Yahoo!
O serviço de busca mais completo da Internet. O que você pensar o Yahoo! encontra.
http://br.busca.yahoo.com/
=
Instruções para entrar na lista, sair da lista e usar a lista em
http://www.mat.puc-rio.br/~nicolau/olimp/obm-l.html
O administrador desta lista é <[EMAIL PROTECTED]>
=


  
  
=
Instruções para entrar na lista, sair da lista e usar a lista em
http://www.mat.puc-rio.br/~nicolau/olimp/obm-l.html
O administrador desta lista é <[EMAIL PROTECTED]>
=


  






Re: [obm-l] trigonometria

2003-02-07 Thread A. C. Morgado




1-2sen^2(x) + sen^4(x) + sen^2(x)
= 1-sen^2(x) + sen^4(x) . Para
dar a resposta do seu gabarito essa expressão deveria valer 1, ou seja, 
sen^2(x) deveria
ser igual a  sen^4(x). 
Seu gabarito está, como sempre, errado.

[EMAIL PROTECTED] wrote:
Olá pessoal, 
 
Como resolver esta: 
 
(UNESP) A expressão [1-2sen^2(x) + sen^4(x) + sen^2(x)]*cos^2 (x) é equivalente
a: 
 
resp: cos^2(x) 





Re: [obm-l] triângulo

2003-02-07 Thread A. C. Morgado




Epa! O angulo dado nao eh C e sim B.

Leahpar Xarm wrote:

  Caro colega esta questão quer ver se vc conhece descaradamente uma forma
de achar a área de um triângulo por (1/2)*a*b*senApha onde a e b são os lados
do triângulo adjacentes ao ângulo apha dado. Temos: 
  [(1/2)AC*BC]*sen(beta)=[7*8*(sqrt3)]/2 
   [EMAIL PROTECTED] wrote: 
  Olá pessoal, 

Veja esta questão: 

(MAUÁ-SP) 

No triângulo ABC, temos: AC= 7m, BC= 8m, beta= ABC=60º. Determine a área
do triângulo. 

resp: 6raiz*(3) ou 10*raiz(3) m^2 

Obs: O triângulo citado é um triângulo de base BC. Eu tentei aplicar a lei
da área [ S=(a.b.sen alfa)/2], mas não é dado o valor de BA. Sendo assim
eu tentei aplicar a lei dos senos para achar BA fazendo 7/sen60º =BA/sen
C daí aparece outra incógnita o sen C. A partir disso eu tentei aplicar a
lei dos cossenos para achar    
o cos C, pois é dado no enunciado AC e BC, para depois calcular o sen C pela
relaçao fundamental sen^2(x) + cos^2 (x)=1, mas não dá para aplicar a lei
dos cossenos, pois não é dado BA. A partir disso entra-se num ciclo vicioso.
Será que não está faltando nem um dado? 
  
  
  Busca Yahoo! 
 O serviço de busca mais completo da Internet. O que você pensar o Yahoo!
encontra. 





Re: [obm-l] RES: [obm-l] triângulo

2003-02-07 Thread A. C. Morgado




So para comentar que, surpreendentemente, o gabarito esta certo.

Eduardo wrote:
  
  
 
  
 
  Olá,
 
   
 
  Bem,  primeiramente você pode aplicar o teorema
dos co-senos a fim de descobrir o lado  não informado, ou seja, o lado AB,
vai cair numa equação do segundo grau de  raízes 5 ou 3. Agora use 1/2absenB
com os dois valores encontrados  anteriormente.
 
   
 
  Espero  ter ajudado.
 
   
 
  Edu
 
 
-Mensagem original-
De:[EMAIL PROTECTED][mailto:[EMAIL PROTECTED]]Em
nome de Johann PeterGustav Lejeune Dirichlet
Enviada em: sexta-feira, 7 de fevereiro de2003 12:17
Para: [EMAIL PROTECTED]
Assunto: Re: [obm-l]triângulo


   

 [EMAIL PROTECTED] wrote:
 

  Olá pessoal, 
  
Veja esta  questão: 
  
(MAUÁ-SP) 
  
No triângulo ABC, temos: AC= 7m, BC= 8m,  beta= ABC=60º. Determine a
área do triângulo. 
  
resp: 6raiz*(3) ou  10*raiz(3) m^2 
  
 
  Deixa eu ver...ce tem
tudo no  triangulo!!O raio e com SLS,certo?a=2Rsen A.Com isso ce
acha o seno de B  pela mesma formula.Ce tem o lado AC e o raio.Para a
area ce precisa de um  angulo que e 180 menos todos os outros.Cabou!!!"!!
 
  
Obs: O triângulo citado é um  triângulo de base BC. Eu tentei aplicar
a lei da área [ S=(a.b.sen alfa)/2],  mas não é dado o valor de BA. Sendo
assim eu tentei aplicar a lei dos senos  para achar BA fazendo 7/sen60º
=BA/sen C daí aparece outra incógnita o sen  C. A partir disso eu tentei
aplicar a lei dos cossenos para achar     
o cos C, pois é dado no enunciado AC e BC, para depois  calcular o sen
C pela relaçao fundamental sen^2(x) + cos^2 (x)=1, mas não dá  para aplicar
a lei dos cossenos, pois não é dado BA. A partir disso entra-se  num
ciclo vicioso. Será que não está faltando nem um dado?

   

   
   Busca Yahoo!

O serviço debusca mais completo da Internet. O que você pensar o Yahoo!
 encontra.






Re: [obm-l] Dúvida

2003-02-09 Thread A. C. Morgado
Bem, primeiro diga-se que ninguem se interessou em responder porque ha 
um evidente erro de digitaçao no enunciado. Em segundo lugar, 
esperava-se que voce dissesse o que encontrou e qual a resposta do seu 
gabarito.
O erro estah nas "primeiras horas". Deve ser "h primeiras horas". Se for 
isso, sua equaçao esyah correta e fornece h=7. O tempo de viagem eh 
h + (h+5) = 19 horas.

elton francisco ferreira wrote:

olá pessoal da OBM-l.

Estou com uma dúvida na questão a seguir...

resolvi eessa questão mas o resultado n bateu com o do
livro. será q vcs podem me ajudar?

Oscar e Douglas fizeram uma viagem de 1862 km. Durante
as primeiras horas, Oscar manteve uma velocidade média
de 86 km/h. Nas seguintes (h + 5) horas, Douglas
dirijiu o carro a uma velocidade média de 105 km/h.
Quanto tempo durou a viagem?

montei a equação desta forma:  86h + 105(h + 5) = 1862




___
Busca Yahoo!
O melhor lugar para encontrar tudo o que você procura na Internet
http://br.busca.yahoo.com/
=
Instruções para entrar na lista, sair da lista e usar a lista em
http://www.mat.puc-rio.br/~nicolau/olimp/obm-l.html
O administrador desta lista é <[EMAIL PROTECTED]>
=


 



=
Instruções para entrar na lista, sair da lista e usar a lista em
http://www.mat.puc-rio.br/~nicolau/olimp/obm-l.html
O administrador desta lista é <[EMAIL PROTECTED]>
=



Re: [obm-l] Problema 01

2003-02-09 Thread A. C. Morgado
As 4h o arco menor eh 1/3 da circunferencia e o maior, 2/3. As respostas 
sao 12,56 e 25,12.

elton francisco ferreira wrote:

A circunferencia de um relógio mede 37,68. Qual é a
mdida do menor arco formado pelos ponteiros de um
relógio as 4h? e do arco maior?

___
Busca Yahoo!
O melhor lugar para encontrar tudo o que você procura na Internet
http://br.busca.yahoo.com/
=
Instruções para entrar na lista, sair da lista e usar a lista em
http://www.mat.puc-rio.br/~nicolau/olimp/obm-l.html
O administrador desta lista é <[EMAIL PROTECTED]>
=


 



=
Instruções para entrar na lista, sair da lista e usar a lista em
http://www.mat.puc-rio.br/~nicolau/olimp/obm-l.html
O administrador desta lista é <[EMAIL PROTECTED]>
=



Re: [obm-l] probabilidade e combinatoria

2003-02-09 Thread A. C. Morgado
amurpe wrote:


Pessoal por favor me ajudem mais uma vez nos seguintes 
problemas.


4)onze cientistas trabalham num projeto sigiloso.
por questoes de segurança , os planos são guardados em 
um cofre protegido por muitos cadeados de modo que só é 
possível abri-los todos se houver pelo menos 5 
cientistas presentes.
a) qual é o numero mínimo possível de cadeados?
b)Na situação do ítem a , quantas chaves cada cientista 
deve ter?


Desde já muito obrigado.


Amurpe

4) Vou dar um espço para quem quiser mais tempo para pensar.
 




























4) Chegam 4 cientistas A, B, C, D. Com as chaves que possuem, abrem 
alguns cadeados, mas nao todos. Existe pelo menos um cadeado que eles 
nao conseguem abrir. Na situaçao do numero minimo de cadeados, existe 
exatamente um cadeado que eles nao conseguem abrir. Batize tal cadeado 
de ABCD. Portanto, ABCD eh o cadeado cuja chave nao estah em poder de A, 
nem de B, nem de C e nem de D. Qualquer outro cientista tem a chave 
desse cadeado, pois esse cientista e A, B, C e D formam um grupo de 5 
cientistas e, portanto, nesse grupo alguem possui a chave. Como o alguem 
nao eh nem A, nem  B, nem C e nem D,...
Analogamente batize os demais cadeados.
Verifique agora que a correspondencia entre cadeados e seus nomes eh 
biunivoca.
O numero de cadeados eh igual ao numero de nomes de cadeados, C(11,4) =330
Cada cientista X possui as chaves dos cadeados que nao possuem X no 
nome, C(10,4) = 210


 



=
Instruções para entrar na lista, sair da lista e usar a lista em
http://www.mat.puc-rio.br/~nicolau/olimp/obm-l.html
O administrador desta lista é <[EMAIL PROTECTED]>
=



Re: [obm-l] probabilidade e combinatoria

2003-02-10 Thread A. C. Morgado




Ha um errinho de digitaçao intermediario no problemados pares. Onde aparece
o produto das combinaçoes deveria aparecer o produto dividido por n!.
A resposta estah certa
Morgado

Cláudio (Prática) wrote:

  Caro Amurpe:

Seguem as minhas soluções para os primeiros três problemas. Vou ter de
pensar um pouco mais sobre o quarto.

  
  
1) Em uma cidade com n+1 habitantes , uma pessoa conta
um boato para uma outra pessoa , a qual por sua vez o
conta para uma terceira pessoa , etc.. . calcule a
probabilidade do boato ser contado m vezes:

a) Sem retornar à primeira pessoa;
b) Sem repetir nenhuma pessoa.


  
  Número de casos possíveis:
Escolha da primeira pessoa (pelo originador do boato) para ouvir o boato: n
Escolha da segunda pessoa (pela primeira a ouvir o boato) para ouvir o
boato: n

Escolha da m-ésima pessoa (pela (m-1)-ésima a ouvir o boato) para ouvir o
boato: n
Total = n^m

a) Número de casos favoráveis:
Escolha do primeiro ouvinte: n
Escolha do segundo: n-1  (estão fora o originador do boato e a primeira
pessoa a ouvi-lo)
Escolha do terceiro: n-1 (estão fora o originador e o segundo ouvinte)
...
Escolha do m-ésimo: n-1
Total = n*(n-1)^(m-1)  ==>  Probabilidade = ((n-1)/n)^(m-1)

b) Número de casos favoráveis:
Escolha do primeiro ouvinte: n
Escolha do segundo: n-1  (estão fora o originador do boato e a primeira
pessoa a ouvi-lo)
Escolha do terceiro: n-2  (estão fora o originador e os dois primeiros
ouvintes)
...
Escolha do m-ésimo: n-m+1 (estão fora o originador e os m-1 ouvintes
anteriores)
Total = n! / (n-m)!  ==>  Probabilidade = (n!/(n-m)!) / n^m

*

  
  
2) Em uma cidade , as pessoas falam a verdade com
probabilidade 1/3.suponha que A faz uma afirmação e que
D diz que C diz que B diz que A falou a verdade.Qual a
probabilidade de A ter falado a verdade ?


  
  
Esse tem cara de pegadinha!

P(A ter falado a verdade) = 1/3, uma vez que cada habitante fala a verdade
com 1/3 de probabilidade.
Em outras palavras, dane-se o que os outros disseram

**

  
  
3) De quantos modos podemos decompor 2n objetos em n
pares ?


  
  Escolha dos primeiros dois objetos: C(2n,2)
Escolha dos dois objetos seguintes: C(2n-2,2)

Escolha dos últimos dois objetos: C(2,2)

Total = C(2n,2)*C(2n-2,2)*...*C(2,2) = (2n)!/(2^n * n!)

Repare que, se após escolher os n pares, nós permutarmos os dois objetos
dentro de cada par (2^n) e, em seguida, permutarmos os n pares (n!),
obteremos o número total de permutações de 2n objetos = (2n)!


Um abraço,
Claudio.

=
Instruções para entrar na lista, sair da lista e usar a lista em
http://www.mat.puc-rio.br/~nicolau/olimp/obm-l.html
O administrador desta lista é <[EMAIL PROTECTED]>
=


  






Re: [obm-l] um problema

2003-02-10 Thread A. C. Morgado
Seja n o numero de candidatos que serao atendidos e seja t o numero de 
dias de atendimento.
9t+34 = n
n=10t
Daih, t=34 e n=340
Os que foram convocados sao 360 e o numero de aprovados eh 3600
A

elton francisco ferreira wrote:

Em um concurso , 1/10 dos aprovados foi selecionado
para entrevista com psicólogos, que deverá ser feita
em 2 dias. Sabendo-se que 20 candidatos desistiram,
não confrimando sua presença para a entrevista, os
psicólogos observaram que, se cada um atendesse 9 por
dia, deixariam 34 jovens sem atendimento.

Para cumprir a meta em tempo hábil, cada um se dispôs,
então, a atender 10 candidatos por dia.
Com base nisso, é correto afirmar que o número de
aprovados no concurso 

a) É múltiplo de 600
b) É divisor de 720
c) É igual a 3400
d) Está compreendido entre 1000 e 3000


___
Busca Yahoo!
O melhor lugar para encontrar tudo o que você procura na Internet
http://br.busca.yahoo.com/
=
Instruções para entrar na lista, sair da lista e usar a lista em
http://www.mat.puc-rio.br/~nicolau/olimp/obm-l.html
O administrador desta lista é <[EMAIL PROTECTED]>
=


 



=
Instruções para entrar na lista, sair da lista e usar a lista em
http://www.mat.puc-rio.br/~nicolau/olimp/obm-l.html
O administrador desta lista é <[EMAIL PROTECTED]>
=



Re: [obm-l] Problema 04

2003-02-10 Thread A. C. Morgado
c, quantidade de notas de cinco; d, quantidade de notas de dez
c+d=10
5c+10d = 65
Daih, d=3 e c=7
Resposta: 21

elton francisco ferreira wrote:


Um caixa automática de um banco só libera notas de R$
5,00 e R$ 10,00. Uma pessoa retirou dessa caixa a
importância de R$ 65,00, recebendo 10 notas. O produto
do número de notas de R$ 5,00 pelo número de notas de
R$ 10,00 é igual a

16 
25
24
21 


___
Busca Yahoo!
O melhor lugar para encontrar tudo o que você procura na Internet
http://br.busca.yahoo.com/
=
Instruções para entrar na lista, sair da lista e usar a lista em
http://www.mat.puc-rio.br/~nicolau/olimp/obm-l.html
O administrador desta lista é <[EMAIL PROTECTED]>
=


 



=
Instruções para entrar na lista, sair da lista e usar a lista em
http://www.mat.puc-rio.br/~nicolau/olimp/obm-l.html
O administrador desta lista é <[EMAIL PROTECTED]>
=



Re: [obm-l] Números complexos

2003-02-10 Thread A. C. Morgado




A primeira. Em A matematica do Ensino Medio, volume 3, voce encontra uma
mini-historia dos complexos.
Morgado

Eduardo wrote:
  
  
 
  
 
  Galera,
estou com  uma dúvida relacionada a números complexos, digamos que  histórica.
 
   
 
   
 
  A primeira
definição  é i^2 =-1 ou a definição foi feita primeiramente para (a; b)x(c;
 d)?
 
   
 
  Abraços
 
   
 
  Edu






[obm-l] Lista paralela e gabaritos errados

2003-02-10 Thread A. C. Morgado




A respeito da criaçao de uma lista paralele e tambem do problema de gabaritos
errados, ja enviei uma mensagem para a lista dando minha opiniao. Naquela
mensagem citava o Claudio, pessoa que, nos ultimos tempos, tem sido a que
mais tem ajudado a dirimir duvidas de nossos companheiros. Vou transcrever
parte de mensagem enviada para mim pelo Claudio, em correspondencia particular.
Desnecessario dizer que o faço com sua autorizaçao e que concordo com todas
as suas palavras. 


 
 
"Só pra deixar claro, minhas opiniões são
as  seguintes:
 
1. A lista deve ser única - se algum problema
não  lhe interessar, simplesmente ignore-o. Além disso, elementar ou avançado
é uma  questão de opinião e de estágio do aprendizado em que a pessoa se
 encontra.
 
2. Quem escreveu ou editou os gabaritos
do Fael  está fazendo um grande mal a quem quer estudar matemática, principalmente
 àqueles que não tem a sorte dele de ter descoberto esta nossa lista. Acho
mesmo  que o Fael deveria escrever para a editora e informá-los sobre todos
os  erros.
 
3. Pra mim, a lista é uma oportunidade de
aprender,  ajudar os outros se for possível e me divertir. Me chateia muito
ver algumas  mensagens mal-humoradas e até mesmo mal-educadas ou arrogantes."
 
 
Abraços a todos.
Morgado




[obm-l] Inscriçao na medalha Fields

2003-02-10 Thread A. C. Morgado




Ha muito tempo alguem perguntou o que significava aquele latinorio da medalha
Fields que aparece ao final de cada mensagem do Dirichlet.
TRANSIRE SVVM PECTVS MVNDOQUE POTIRE
   
CONGREGATI EX TOTO ORBE MATHEMATICI OB SCRIPTA INSIGNIA TRIBVERE
   
Fields Medal(John Charles Fields)

 
  

Bem, supondo que seja POTIRI em vez de POTIRE, 
a traducao literal eh:
 
"Ultrapassar o próprio peito e dominar o
mundo;  
 
outorgado pelos matematicos de todo o mundo
 reunidos, em funcao dos destacados escritos".
 
 
 
De uma maneira bem livre:
 
"Superar os limites da inteligencia e conquistar
o  universo;
 
premio outorgado pela congregacao de todos
os  matematicos do mundo, em funcao da destacada obra do premiado"

 
A
traduçao eh do Professor Jose Paulo Carneiro, ex-membro destacado desta lista,
que dela se afastou por nao conseguir suportar algumas manifestaçoes de mau
humor de alguns membros.
   
  
  
  
   
 .  






Re: [obm-l] Inscriçao_na_medalha_Fields

2003-02-11 Thread A. C. Morgado




A medalha Fields eh uma medalha para matematicos jovens. Ha um limite de
idade. 

Johann Peter Gustav Lejeune Dirichlet wrote:

  Ja faz um certo tempo sim.Esta traduçao esta na RPM que fala da Fields.Falando
nisso ja perceberam que e sempre um frances,um estadunidense ou um ingles
que ganha Fields?Ha pouquissimas exceçoes,algumas famosas como o Atle Selberg(alias
acho que o Paul Erdös devia ter ganho em conjunto com a demonstraçao elementar
do TNP). 
   "A. C. Morgado" <[EMAIL PROTECTED]> wrote: 
  
  Ha
muito tempo alguem perguntou o que significava aquele latinorio da medalha
Fields que aparece ao final de cada mensagem do Dirichlet.
 
TRANSIRE SVVM PECTVS MVNDOQUE POTIRE
 
CONGREGATI EX TOTO ORBE MATHEMATICI OB SCRIPTA INSIGNIA TRIBVERE
 
Fields Medal(John Charles Fields)

 
 Bem, supondo que seja POTIRI
em vez de POTIRE,   
a traducao literal eh:
 
"Ultrapassar o próprio peito e dominar
o mundo; 
 
outorgado pelos matematicos de todo
o mundo reunidos, em funcao dos destacados escritos".
 
 
 
De uma maneira bem livre:
 
"Superar os limites da inteligencia
e conquistar o universo;
 
premio outorgado pela congregacao de
todos os matematicos do mundo, em funcao da destacada obra do premiado"

 
A
traduçao eh do Professor Jose Paulo Carneiro, ex-membro destacado desta lista,
que dela se afastou por nao conseguir suportar algumas manifestaçoes de mau
humor de alguns membros.
  
 
  
  
 
   . 


  
  
  
  Busca Yahoo! 
 O serviço de busca mais completo da Internet. O que você pensar o Yahoo!
encontra. 





Re: [obm-l] Observar passos para progredir

2003-02-28 Thread A. C. Morgado
Meu caro,
em primeiro lugar, quero agradecer ter sido posto, ainda que 
imerecidamente, em tão honrosas companhias; sao todos, alem de grandes 
matematicos e professores, pessoas de carater e estimados amigos.
Uma coisa que me enche de orgulho eh  ter sido professor de tres dos 
citados (Jose Paulo Carneiro, Paulo Cezar Carvalho e Eduardo Wagner), 
hoje matematicos brilhantes, na epoca alunos dos quais se dizia que so 
erraram uma unica vez: quando pensaram que haviam cometido um pequeno 
engano, mas eh claro que nao tinham.
Muito pensei se devia escrever uma resposta ou se devia cair na real e 
perceber que estava sendo citado apenas por delicadeza sua que citara 
pessoas com as quais havia trabalhado em muitos projetos. Entretanto, 
resolvi escrever esta mensagem porque ela me permite fazer uma confissao 
e um agradecimento. Eu sou uma pessoa de muita sorte. Nunca tive 
professores bons; todos os professores que tive eram otimos. Nunca tive, 
no primario, uma professora de matematica que nao gostasse de 
matematica. No antigo admissao (seria equivalente a algo entre a quarta 
e a quinta series), fui aluno de um excelente professor, Serafim 
Rodrigues Morgado (apesar do sobrenome, nao era meu parente), que foi 
professor tambem de pelo menos mais um membro da lista, o Jose Francisco 
Guimaraes Costa, e a quem devo a transformaçao do meu nome de guerra de 
Augusto Cesar para Morgado. No ginasio, foi aluno de um engenheiro, 
Claudionor Teixeira Braga, que me ensinou muito e, principalmente, fez 
com que eu gostasse muito de Matematica. No cientifico (equivalente ao 
ensino medio de hoje em dia), o Professor Delamare, tambem engenheiro e 
muito brincalhao (era pai do Julio Delamare, jornalista esportivo que 
dah nome ao parque aquatico do Maracana), tambem excelente professor. 
Depois, fui aluno de Jacob Palis, de Celio Pinto de Almeida, de Leon 
Clement Rousseau, de Alberto (a idade me impede de recordar o sobrenome 
dele, mas nao eh o Azevedo, que eh muito conhecido; o Alberto, embora 
excelente professor, preferiu se dedicar a engenharia), de Luiz 
Rodrigues Loureiro, de Jorge Alberto Barroso, de Oton Nogueira. Fora do 
ensino regular, tive cursos com o Elon, com Mauricio Matos Peixoto, com 
Hilton Machado, com Imre Simon, com Leopoldo Nachbin, com Pedro 
Fernandez. Ate fora da Matematica minha sorte continuou. Tive excelente 
professor de Portugues (o Padre Antonio Malheiros, brilhante orador e 
responsavel por muitos dos discursos de Getulio Vargas, discursos tao 
bons que conduziram Vargas a Academia Brasileira de Letras), de Fisica 
(por coincidencia, dois astronomos e diretores do Observatorio Nacional 
e do Observatorio do Valongo: Muniz Barreto e Luiz Machado; alem de 
Jader Bennuzzi Martins, do CBPF), de Ingles (o Professor Blum, que tinha 
um programa na TV Tupi e foi um precursor do ensino de idiomas por meio 
de musicas, alem de pai da Norma Blum, atriz "idola"  dos adoscelentes 
da minha epoca), de Espanhol (o Professor Walter). Ate de Historia, o 
Professor Herminio (esse eh um caso curioso; odiava dar aulas, dar aulas 
era para ele um meio de ganhar um dinheirinho enquanto nao se formava em 
Direito; mas, alem de pessoa extremamente culta, era um excelente 
professor; todos adoravam suas aulas, exceto ele!) .
O Jose Paulo Carneiro, quando me citava, dizia:  ... o Morgado, com sua 
proverbial sorte, ...
Pois eh, o Jose Paulo tem toda razao!
Por que meus professores foram tao importantes para mim? Pelo exemplo, 
pela cultura, pela orientaçao, pelas corretas indicaçoes do que ler.
Quanto aos anos-luz, essa eh a distancia que me encontro dos outros 
citados. 
Um forte abraço.
Morgado.

[EMAIL PROTECTED] wrote:

Queridos professores e amigos,

 É  fato  que homens seguem passos de outros homens, que alunos seguem

 os  de  seus mestres até aonde podem, muitas vezes, ir. Raras vezes, pode

 ocorrer  que alunos igualem-se a seus mestres, para os substituir, já que

 a vida flui, neste planeta.

   Fui aluno do Ralph, em 1989, na turma IME/ITA do Impacto.  Cursei o

 IME  de  90  a  94.  Fiz  mestrado  no  mesmo  Instituto  de 98 a 99. Sem

 idolatria,  afirmo  admirar  os passos de Pitombeira de Carvalho, Nicolau

 Corsão Saldanha, Augusto César de Oliveira Morgado, Ralph Costa Teixeira,

 João  Bosco, Paulo Cezar Pinto Carvalho, Pedro Fernandez, Eduardo Wagner,

 Élon Lages Lima, João Paulo Q. Carneiro, Carlos Gustavo T. de A. Moreira.

 Graças  a  Deus! A Internet permite-me ser aluno, à distância, dos demais

 citados, os quais não conheci pessoalmente.

 Também,  vale  a  penas destacar as contribuições do melhores alunos,

 como,  por  exemplo,  Cláudio  ([EMAIL PROTECTED]), que mui

 contribui  para desenvolvimento dos alunos mais retardatários, dentro dos

 quais, incluo-me, com naturalidade.

   Nesta  seqüência  de  observar passos para o progresso, nós eternos

 alunos  gostaríamos  de  saber,  se  vocês  professores conheceram outros

 mestres   por   vós  admiráveis  e  

[Fwd: Re: [obm-l] verificações]

2003-03-02 Thread A. C. Morgado




 

  

  
  
  
  


  
  
  
  


  
  
  
  


  
  
  
  


  
  
  
  

  

Esta mensagem estah sendo reenviada para corrigir um erro de soma (estava
2+3 = 6 ).

a)(1,2)*(3,7) = (8, 5)
(3,7)*(1,2) = (5, 8) 

b)[(1,2)*(3,7)]* (2,4) = (8, 5)*(2,4) = (12,7)
(1,2)*[(3,7)* (2,4)] = (1, 2)*(7,9) = (10,9)


Em Sat, 1 Mar 2003 16:33:37 -0300 (ART), pichurin  disse:

> Mostre que no R^2 , a operação (a,b)*(c,d)=(a+d,b+c)
> não verifica as propriedades comutativa e associativa
> da adição.
> 
> ___
> Busca Yahoo!
> O serviço de busca mais completo da Internet. O que você pensar o Yahoo! encontra.
> http://br.busca.yahoo.com/
> =
> Instruções para entrar na lista, sair da lista e usar a lista em
> http://www.mat.puc-rio.br/~nicolau/olimp/obm-l.html
> O administrador desta lista é 
> =
> 
> 

=
Instruções para entrar na lista, sair da lista e usar a lista em
http://www.mat.puc-rio.br/~nicolau/olimp/obm-l.html
O administrador desta lista é 
=







[obm-l] Problemas em aberto IV

2003-03-02 Thread A. C. Morgado
O Luís Lopes mandou ha algum tempo:
Prove que
tan(3*Pi/11) + 4*sin(2*Pi/11) = sqrt(11).
Embora eu tenha uma ideia muito clara do que fazer (usar trigonometria 
do tempo dos gregos, isto eh, construir um conveniente quadrilatero 
inscrito e aplicar o teorema de Ptolomeu), quando tentei nao consegui. 
Eh verdade que nao pude dedicar a esse problema o tempo que ele parece 
exigir.
Morgado

=
Instruções para entrar na lista, sair da lista e usar a lista em
http://www.mat.puc-rio.br/~nicolau/olimp/obm-l.html
O administrador desta lista é <[EMAIL PROTECTED]>
=


Re: [obm-l] Combinações

2003-03-02 Thread A. C. Morgado






pichurin wrote:

  Observe:
tome a e b como números reais.
a*5*tg^2(x) + b*2/cos^2(x)= a*5*sen^2(x)/cos^2(x) +
b*2/cos^2(x)=(a*5*sen^2(x) + b*2)/cos^2(x)
Então, a= (3*cos^2(x) -2*b)/(5*sen^2(x))
Isso significa que a depende de x.

NAO SIGNIFICA NAO. (3*cos^2(x) -2*b)/(5*sen^2(x)) = [3 - 3 SEN^2(X) - 2B]/5SEN^2(X)
= [(3-2B) - 3SEN^2(X)]/5SEN^2(X)
Se 3 - 2b = 0,  

  
Isso influi em alguma coisa?Explique.

 --- Augusto Cesar de Oliveira Morgado
<[EMAIL PROTECTED]> escreveu: > 1+ (tan^2)(x) =
(sec^2)(x)
  
  

Em Sun, 2 Mar 2003 13:20:36 -0300 (ART), pichurin
<[EMAIL PROTECTED]> disse:



  Como vc chegou neste resultado?



 --- Augusto Cesar de Oliveira Morgado
<[EMAIL PROTECTED]> escreveu: > sim,
  

(3/2)h(x) -


  (3/5)g(x) = 3
  
  

Em Sat, 1 Mar 2003 16:46:18 -0300 (ART),

  

pichurin


  
<[EMAIL PROTECTED]> disse:



  Em F(-pi/2,pi/2), verifique se a função f
  

constante e


  igual a 3 é combinação linear de g e h
  

  

definidas


  
por


  g(x)=5tan^(2)(x) e h(x)=2/(cos^(2)(x)).


  

  

  
  ___
  
  

  

  Busca Yahoo!
O serviço de busca mais completo da Internet.
  

  

O


  
que você pensar o Yahoo! encontra.


  http://br.busca.yahoo.com/

  

  

  
  =
  
  

  

  Instruções para entrar na lista, sair da lista
  

  

e


  
usar a lista em

  

http://www.mat.puc-rio.br/~nicolau/olimp/obm-l.html


  

  O administrador desta lista é
  

<[EMAIL PROTECTED]>

  

  
  =
  
  

  

  
  



  

  
  =
  
  

  
Instruções para entrar na lista, sair da lista e
usar a lista em


  

http://www.mat.puc-rio.br/~nicolau/olimp/obm-l.html


  
O administrador desta lista é
<[EMAIL PROTECTED]>


  

  
  =
  
  

  
  

  
  ___
  
  

  Busca Yahoo!
O serviço de busca mais completo da Internet. O
  

que você pensar o Yahoo! encontra.


  http://br.busca.yahoo.com/

  

  
  =
  
  

  Instruções para entrar na lista, sair da lista e
  

usar a lista em

http://www.mat.puc-rio.br/~nicolau/olimp/obm-l.html


  O administrador desta lista é
  

<[EMAIL PROTECTED]>

  
  =
  
  

  
  



  
  =
  
  
Instruções para entrar na lista, sair da lista e
usar a lista em
http://www.mat.puc-rio.br/~nicolau/olimp/obm-l.html
O administrador desta lista é
<[EMAIL PROTECTED]>


  
  = 

___
Busca Yahoo!
O serviço de busca mais completo da Internet. O que você pensar o Yahoo! encontra.
http://br.busca.yahoo.com/
=
Instruções para entrar na lista, sair da lista e usar a lista em
http://www.mat.puc-rio.br/~nicolau/olimp/obm-l.html
O administrador desta lista é <[EMAIL PROTECTED]>
=


  






Re: [obm-l] análise combinatória I

2003-03-03 Thread A. C. Morgado






  Ha tres tipos de retas:
 1) a reta dos 5 pontos
 2) retas determinadas´por um dos 5 pontos e um dos outros 7; essas sao em 
numero de 5x7=35
 3) retas determinadas por dois dos 7 pontos; essas sao em numero de C(7,2) 
= 21.
 A resposta eh 1+35+21=57.
 Seu gabarito, como sempre estah errado.
 Faça um favor a todos nos e, principalmente, a voce. Jogue fora esse fasciculo 
e leia um bom livro.
 Outra soluçao:
 Para formar uma reta voce deve escolher 2 dos 12 pontos, o que pode ser
feito de C(12,2)=66 modos. Nessa brinacadeira a reta dos 5 pontos foi contado
C(5,2)=10 vezes. Retirando-a, a contagem baixa para 66 - 10 = 56. Mas aih
nos exageramos pois retiramos essa reta todas as vezes que ela foi contada
e uma vez ela deve ser contada. A resposta eh 57.
 Esse erro mostra que o autor dos seus fasciculos eh muito ruim de Combinatoria. 
Nesses fasciculos aprende-se tanto quanto em aula de futebol dada pelo Junior 
Baiano.
 Morgado
 [EMAIL PROTECTED]
wrote:
 
Olá pessoal, 
  
 Como resolver esta: 
  
 (UF. UBERLÂNDIA) Em um plano há 12 pontos, dos quais três nunca são colineares, 
exceto 5 que estão sobre uma mesma reta. O número de retas determinadas por 
esses pontos é: 
  
 resp: 56 
   
 
 




Re: [obm-l] Uma de combinatória e duas de binômios

2003-03-04 Thread A. C. Morgado





   O problema 2 nao tem sentido algum. Todos os termos do desenvolvimento
sao termos em x^20. Como ordena-los segundo as potencias decrescentes de
x?
 Fael, jah que voce nao vai mesmo jogar fora esses seus fasciculos, pelo
menos entrega para a gente o nome dos autores!
 Morgado
  
 [EMAIL PROTECTED]
wrote:
 
Olá pessoal, 
  
 Como resolver estas: 
  
 (EESCUSP) O número de combinações de n elementos, p a p, que contém k elementos 
determinados é: 
  
 resp: C_n-k,p-k 
 obs: Eu não cheguei a resposta por não entender o que o examinador esta
querendo dizer com "k elementos determinados". 
  
 (UF.UBERLÂNDIA) Desenvolvendo-se o binômio ( 2x^2 + [x/2]^2 )^10 segundo 
as potências decrescentes de x, o 6º termo será: 
  
 obs: Eu apliquei a fórmula do termo geral e cheguei ao resultado de T_6= 
252x^37. Se chegarem no mesmo resultado não precisam resolver. Mas se chegarem 
a uma resposta diferente me digam, por favor, o por quê. 
  
 Mais uma questão de binômios de Newton: 
  
 No desenvolvimento de (3x + 13)^n há 13 termos. A soma dos coeficientes
destes termos é igual a: 
  
  
  
  
   
 
 




Re: [obm-l] Uma de combinatória e duas de binômios

2003-03-04 Thread A. C. Morgado




Traduzindo a primeira::
O número de combinações de n
elementos, 1, 2,..., n, tomadas p a p, que contêm k elementos determinados,
1, 2,..., k,  é: 

Resolvendo a ultima:
n+1=13
n=12
O binomio eh
(3x+13)^12
A soma dos coeficientes de x eh  (3*1+13)^12 = 16^12.

[EMAIL PROTECTED] wrote:
Olá pessoal, 
 
Como resolver estas: 
 
(EESCUSP) O número de combinações de n elementos, p a p, que contém k elementos
determinados é: 
 
resp: C_n-k,p-k 
obs: Eu não cheguei a resposta por não entender o que o examinador esta querendo
dizer com "k elementos determinados". 
 
(UF.UBERLÂNDIA) Desenvolvendo-se o binômio ( 2x^2 + [x/2]^2 )^10 segundo
as potências decrescentes de x, o 6º termo será: 
 
obs: Eu apliquei a fórmula do termo geral e cheguei ao resultado de T_6=
252x^37. Se chegarem no mesmo resultado não precisam resolver. Mas se chegarem
a uma resposta diferente me digam, por favor, o por quê. 
 
Mais uma questão de binômios de Newton: 
 
No desenvolvimento de (3x + 13)^n há 13 termos. A soma dos coeficientes destes
termos é igual a: 
 
 
 
 
  





Re: [obm-l] RE: [obm-l] Uma de combinatória e duas de binômios

2003-03-04 Thread A. C. Morgado




Houve um evidente errinho de digitaçao no final da soluçao da terceira questao.
Onde estah 15 leia-se 16.

Artur Costa Steiner wrote:
   
  
   
  
 
  
 
  
 
  
  

  

   
 
  Na primeira pergunta, k elementos
determinados significa k elementos fixados, ou especificados. Por exemplo,
com os 10 priemeiros números naturais, quamtas combinações simples de 4 elementos
existem nas quais os números 3 e 5 aparecem? Neste caso, determinamos os
números 3 e 5 e só estamos interessados nas combinações em que eles aparecem.
  
  
  No caso geral, a resposta  é de fato dada por  C_n-k,p-k. Para ver isto, observe
que o número de combinações nas quais os k elementos fixos comparecem são
obtidas fixando-se tais elementos e colocando junto deles os demais n-k,
combinados agora p-k a p-k, visto que vc não vai colocar de novo os elementos
determinados.
  
   
  
  Não 
pude ver agoara a segunda questão. Mas no caso da terceira, como temos 13 
termos e há n+1 termos, segue-se que n=12. para obtermos a soma dos coeficientes
basta fazer x=1, obtendo (3+13)^12 = 15^12. 
  
   
  
  Olá pessoal, 
 
 Como resolver estas: 
 
 (EESCUSP) O número de combinações de n elementos, p a p, que contém k elementos 
determinados é: 
 
 resp: C_n-k,p-k 
 obs: Eu não cheguei a resposta por não entender o que o examinador esta querendo
dizer com "k elementos determinados". 
 
 (UF.UBERLÂNDIA) Desenvolvendo-se o binômio ( 2x^2 + [x/2]^2 )^10 segundo
as potências decrescentes de x, o 6º termo será: 
 
 obs: Eu apliquei a fórmula do termo geral e cheguei ao resultado de T_6= 
252x^37. Se chegarem no mesmo resultado não precisam resolver. Mas se chegarem 
a uma resposta diferente me digam, por favor, o por quê. 
 
 Mais uma questão de binômios de Newton: 
 
 No desenvolvimento de (3x + 13)^n há 13 termos. A soma dos coeficientes destes
termos é igual a: 
 
 
 
 
  
  
  





Re: [obm-l] retorno da questão de binômios

2003-03-04 Thread A. C. Morgado




A resposta eh 252* (x^15)

[EMAIL PROTECTED] wrote:
Olá pessoal, 
 
Houve um erro de digitação quando enviei e, na verdade, não é (2x^2 + [x/2]^2
)^10 , mas sim (2x^2 + [x/2] )^10  
 
 
(UF.UBERLÂNDIA) Desenvolvendo-se o binômio (2x^2 + [x/2] )^10 segundo as
potências decrescentes de x, o 6º termo será:  
 
obs: Eu apliquei a fórmula do termo geral e cheguei ao resultado de T_6=
252x^37. Se chegarem no mesmo resultado não precisam resolver. Mas se chegarem
a uma resposta diferente me digam, por favor, o por quê. 





Re: [obm-l] geometria analítica

2003-03-06 Thread A. C. Morgado




1) (B+C)/2 = (3,1)  ;    (A+C)/2 = (0,5)  ;        (A+B)/2 =  (2,3)
    B+C = (6,2)        ;     A+C = (0, 10)    ;        A+B = (4,6)
   2(A+B+C) = (10; 18)     ;     A+B+C = (5, 9)
    A = ( - 1, 7)     ;   B= (5, - 1)       ;   C = (1, 3) 
y - 3 = [(3 - -1) / (1 - 5)] (x - 1)
y - 3 = - (x-1)
y + x = 4

2) r:  x+y =1
s:  3x - 2y + 6 = 0.
Resolvendo o sistema,  x =  - (4/5)      y =  9/5

[EMAIL PROTECTED] wrote:
Olá pessoal,  
 
Como resolver estas: 
 
  
(FAAP-SP) Sejam as 0xy um sistema cartesiano ortogonal e o triângulo ABC,
para o qual os pontos M_A= (3,1), M_B= (0,5) e M_C= (2,3) são os pontos médios
dos lados, opostos aos vértices A, B e C, respectivamente. Pede-se : 
a) achar as coordenadas do vértice A 
b) achar a equação da reta suporte do lado BC 
 
resp:  
a) A(-1;7) 
b) x + y -4= 0 
 
(UFRS) As retas r e s da figura interceptam-se no ponto de ordenada: 
 
resp: 9/5 
 
obs: Na figura a reta s está sobre os pontos (-2;0) e (0,3). Já a reta r
está sobre os pontos (1;0) e (0;1) 
  





Re: [obm-l] (O (sqrt n))

2003-03-06 Thread A. C. Morgado
f(n) = O(g(n)) means there are positive constants c and k, such that 
0<=  f(n)<=  cg(n) for all n >= k. The values of c and k must be fixed 
for the function f and must not depend on n.



Henrique P. Sant'Anna Branco wrote:

Quando eu mando o Maple fazer uma série de Taylor para uma função, aparece
esse O também.
Creio eu que seja algo muito pequeno, um infinitesimal, não sei direito.
Henrique.

- Original Message -
From: Wagner
To: [EMAIL PROTECTED]
Sent: Wednesday, March 05, 2003 4:34 PM
Subject: [obm-l] (O (sqrt n))
Oi para todos !

Estava vendo a sequência A006218 no
http://www.research.att.com/~njas/sequences/
e me deparei com O(sqrt(n)) na fórmula da sequência
Se alguém puder me esclarecer o que isso quer dizer
eu agradeceria muito.
André T.

=
Instruções para entrar na lista, sair da lista e usar a lista em
http://www.mat.puc-rio.br/~nicolau/olimp/obm-l.html
O administrador desta lista é <[EMAIL PROTECTED]>
=
 



=
Instruções para entrar na lista, sair da lista e usar a lista em
http://www.mat.puc-rio.br/~nicolau/olimp/obm-l.html
O administrador desta lista é <[EMAIL PROTECTED]>
=


Re: [obm-l] Re: [obm-l] Reta de Euler

2003-03-06 Thread A. C. Morgado




Eles (baricentro, ortocentro e, como voce bem corrigiu, circuncentro) sempre
se alinham.
Morgado

[EMAIL PROTECTED] wrote:

  Numero um:Esses tres pontos nao necessariamente se alinham.Se nao me engano
GIH e obtuso.Na verdade e circuncentro e nao incentro.
Numero Dois:Uma demonstraçao esta na Eureka,antes do numero 4.E bem simples:considere
a mediana relativa a um lado e os pontos notaveis ,e use semelhança de triangulos.Prove
antes que o segmento que liga um ponto ao ortocentro e o dobro do que liga
o circuncentro ao medio do oposto.
Outro jeito e usar coordenadas baricentricas ou vetores.Veja no artigo da
RPM do Morgado,ou pergunte pro proprio. 
-- Mensagem original --

  
  
Alguém poderia me ajudar na seguinte demonstração:

   "Os pontos notáveis - baricentro, incentro e o 
ortocentro - são sempre colineares".

 Desde já agradeço!


__
E-mail Premium BOL
Antivírus, anti-spam e até 100 MB de espaço. Assine já!
http://email.bol.com.br/


=
Instruções para entrar na lista, sair da lista e usar a lista em
http://www.mat.puc-rio.br/~nicolau/olimp/obm-l.html
O administrador desta lista é <[EMAIL PROTECTED]>
=


  
  
TEA WITH ME THAT I BOOK YOUR FACE


--
Use o melhor sistema de busca da Internet
Radar UOL - http://www.radaruol.com.br



=
Instruções para entrar na lista, sair da lista e usar a lista em
http://www.mat.puc-rio.br/~nicolau/olimp/obm-l.html
O administrador desta lista é <[EMAIL PROTECTED]>
=


  






Re: [obm-l] Probabilidade

2003-03-08 Thread A. C. Morgado
Vamos chamar as moedas de normais e defeituosa.
P(defeituosa na certeza de 4 caras) = P(defeituosa e 4 caras)/P(4 caras)
P(defeituosa e 4 caras) = P(defeituosa)*P(4caras na certeza de 
defeituosa) = (1/4)*1 = 1/4
P(4 caras) = P(4 caras e defeituosa  OU  4 caras e normal) =
= P(defeituosa e 4 caras) + P(normal e 4 caras) =
= 1/4 + P(normal)*P(4caras na certeza de normal) = 1/4 + 3/4 * [(1/2)^4] 
= 19/64
A resposta eh  (1/4)/(19/64) = 16/19

cfgauss77 wrote:

Gostaria de uma ajudinha no seguinte problema:

 "Numa urna temos quatro moedas, sendo que uma dela é 
defeituosa apresentando duas caras em suas faces. Alguém 
escolhe aleatoriamente uma dessas moedas e faz quatro 
arremessos sucessivos conseguindo quatro caras. 
Determine qual a probabilidade dessa moeda ser a 
defeituosa".



__
E-mail Premium BOL
Antivírus, anti-spam e até 100 MB de espaço. Assine já!
http://email.bol.com.br/
=
Instruções para entrar na lista, sair da lista e usar a lista em
http://www.mat.puc-rio.br/~nicolau/olimp/obm-l.html
O administrador desta lista é <[EMAIL PROTECTED]>
=
 



=
Instruções para entrar na lista, sair da lista e usar a lista em
http://www.mat.puc-rio.br/~nicolau/olimp/obm-l.html
O administrador desta lista é <[EMAIL PROTECTED]>
=


Re: [obm-l] Dúvidas??

2003-03-08 Thread A. C. Morgado




1) Embora, como um quase sexagenario, nao aprecie muito essas coisas, acho
que i. e. eh uma abreviatura para isto eh e tq eh uma abreviatura para tal
que. 
3) Vou ser bem intuitivo (isto eh, nao vou provar coisas que deveria, mas
que sao muito plausiveis)

?Quantas vezes em media voce deve jogar um dado honesto para obter um 4?
\a probabilidade de obter um 4 em um lançamento eh 1/6, i. e., em media voce
demora 6 lançamentos para obter um 4.
O tempo medio para completar um album com n figurinhas eh a soma do tempo
medio? gasto para encher uma casa do album, com o tempo gasto a partir daih
para encher uma segunda casa, etc.
A prob. de uma figurinha encher uma casa do album eh n/n; depois disso, a
prob. de uma figurinha encher uma segunda casa (i.e., nao ser duplicata da
primeira) eh (n-1)/n, etc. 
A resposta eh ? n/n ?+ ?n/(n-1) + ... + n/1 = ?n*(1 + 1/2 + ... + 1/n).
A soma entre parenteses eh bem aproximada, para n grande, por ln(n). O erro
tende, quando n tende a infinito, a um numero compreendido entre 0,5 e 0,6
que eh representado por um gama minusculo e chamado de constante de Euler-Mascheroni.
Ateh hoje nao se sabe se essa constante eh racional ou irracional

Leonardo Borges Avelino wrote:
  
  
 
  
 

  1) A primeira dúvida é sobre o significado
das  seguintes simbologias:
 
  ?
 
  >i.e.
 
  >deg(x)
 
  >tq
 
  ?
 
  2) Suponhamos um álbum com n figurinhas,
e que  compramos 1 figurinha por dia. Por que o nº esperado de dias para
que  esse?álbum fique?completo é n*ln(n) ??? Como demonstrar  isto??
 
  ?
 
  3) Pergunto aos amigos o que acham dos
livros:  
 
  ?
 
  >Problemas de Aritmética ( Comte. Paulo
Pessoa)  
 
  >Problemas de Algebra ( Comte. Paulo
Pessoa)  
 
  >Exercícios de Geometria Plana ( Edgar
de  Alencar Filho) 
 
  >Problemas Selecionados de Matemática
- Volume I  (Antonio Luiz do Santos e Raul F. W. Agostinho) 
 
  ?
 
  ?
 
  ?
 
  ?
 
  Valeu!!
 
  Leonardo Borges  Avelino.?






Re: [obm-l] AJUDA COM LIMITES

2003-03-11 Thread A. C. Morgado




3) (1-cosx)/x^2. Multiplique em cima e em baixo por 1+cosx.
Fica  sen^2(x)/[x^2} * 1/(1+cosx). O primeiro fator tende a 1; o segundo,
a 1/2.
Resposta: 1/2
2) Divida em cima e em baixo por raiz(x).
Fica   raiz ( 1 +  1/x)  /  raiz (9 + 1/x) cujo limite eh   raiz (1+0)/raiz(9+0)
 = 1/3

Afemano wrote:
  
  
 
  
 

  Olá, alguém pode me ajudar com esse problemas
 "simples" ??
 
   
 
  1)  lim(x-> -3)  (raiz(x^2 +  16) -
5 )/ ( x^2 - 3x )
 
   
 
  2) lim(x-> +00)  ( raiz(x + 1) ) /
(  raiz(9x + 1) )
 
   
 
  3) lim(x-> 0) ( 1 - cosx ) / x^2
 
   
 
  Valeu ae !!
 
  gabriel






Re: [obm-l] AJUDA COM LIMITES

2003-03-11 Thread A. C. Morgado




1) O primeiro eh isso mesmo? Se for nao ha dificuldade alguma. O numerador
tende a 0 e o denominador tende a 18. Resposta: 0/18 = 0  
Afemano wrote:
  
  
 
  
 

  Olá, alguém pode me ajudar com esse problemas
 "simples" ??
 
   
 
  1)  lim(x-> -3)  (raiz(x^2 +  16) -
5 )/ ( x^2 - 3x )
 
   
 
  2) lim(x-> +00)  ( raiz(x + 1) ) /
(  raiz(9x + 1) )
 
   
 
  3) lim(x-> 0) ( 1 - cosx ) / x^2
 
   
 
  Valeu ae !!
 
  gabriel






Re: [obm-l] AJUDA COM LIMITES

2003-03-11 Thread A. C. Morgado




Um errinho na ultima digitaçao. Deveria ser 
lim_x->0 1/2 * cos(x)) = 1/2 * cos(0) = 1/2.



Henrique P. Sant'Anna Branco wrote:

  - Original Message -
From: Afemano
To: [EMAIL PROTECTED]
Sent: Tuesday, March 11, 2003 4:59 PM
Subject: [obm-l] AJUDA COM LIMITES

  
  
1)  lim(x-> -3)  (raiz(x^2 + 16) - 5 )/ ( x^2 - 3x )

  
  
sqrt((-3)^2 + 16) > 0 e (-3^2 - 3*(-3)) diferente de zero, então é só
substituir x = -3.
Resp.: 0

  
  
2) lim(x-> +inf)  ( raiz(x + 1) ) / ( raiz(9x + 1) )

  
  
Reescrevendo a expressão acima temos lim_x-> +inf (sqrt(1 + 1/x) / sqrt(9 +
1/x). Como lim_x-> 1/x = 0, temos que lim_x-> +inf (sqrt(1 + 1/x) / sqrt(9 +
1/x) = 1/3.

  
  
3) lim(x-> 0) ( 1 - cosx ) / x^2

  
  
Usando L'Hôpital duas vezes, temos lim_x->0 1/2 * cos(x)) = 0.

Abraços,
Henrique.

=
Instruções para entrar na lista, sair da lista e usar a lista em
http://www.mat.puc-rio.br/~nicolau/olimp/obm-l.html
O administrador desta lista é <[EMAIL PROTECTED]>
=


  






Re: [obm-l] RE: [obm-l] geometria analítica

2003-03-11 Thread A. C. Morgado




Um errinho de conta!
Onde esta mod(m) = 1/2 deveria estar mod(m) = 1/sqrt(3).
Daih, seguir-se-ia
tg^2(alfa) + 1
= sec^2(alfa) => sec^2(alfa) = 4/3 => sen^2(alfa) = 1/4. => sen(alfa)
= 1/2. 


leandro wrote:
   
  
   
  
  
  

  Fael,
  
   
  
  No
numero 1) eu substitui o valor y=mx na equacao da circunferencia e dai voce
encontra a seguinte equacao do 2o grau 
  
   
  
  (m^2+1)x^2
– 8x + 12 = 0. Como foi dito que m > 0, entao temos que a intersecao da
reta com a circunferencia deve produzir somente 1 ponto, portanto, fazendo
o discriminante da equacao encontrada igual a zero a gente encontra 
  
   
  
  64
– 4(12)(m^2+1) = 0 => mod(m) = ½ = > m=1/2 (pois m > 0). Agora,
m e o coeficiente angular da reta , ou seja, e a tangente do angulo que estamos
querendo encontrar. O valor do seno do angulo pode ser encontrado pela formula
  
  
   
  
  tg^2(alfa) + 1
= sec^2(alfa) => sec^2(alfa) = 5/4 => sen^2(alfa) = 1/5. => sen(alfa)
= 1/sqr(5). 
  
   
  
  Eu
nao encontrei esse resultado ½ que voce me forneceu. Sera que errei em algum
lugar ? 
  
   
  
  Leandro.
  
  
   
  
  -Original
Message-
 From: [EMAIL PROTECTED]
[mailto:[EMAIL PROTECTED]] On Behalf Of [EMAIL PROTECTED]
 Sent: Tuesday, March  11, 2003 1:27 PM
 To: [EMAIL PROTECTED]
 Subject: [obm-l] geometria 
analítica
  
   
  
  Olá Morgado,
  
 
 Como resolver estas: 
 
 
 (FUVEST) A reta y= mx (m>0) é tangente à circunferência (x-4)^2 + y^2=4. 
Determine o seno do ângulo que a reta forma com o eixo x. 
 
 resp: 1/2 
 
 (U.E. Londrina) Sejam a circunferência (lambda) x^2 + y^2 + 2x - 4y +1=0
e o ponto P(-1,4) pertencente a lambda. A equação da reta tangente lambda
pelo ponto P é: 
 
 resp: y=4 
  
  





Re: [obm-l] fração

2003-03-11 Thread A. C. Morgado




A soma vale 1+ 9/40* = 1 + 4/40 + 5/40 = 1 + 1/10 + 1/8.
* devemos remover dois termos que somados deem 9/40. Um dos termos tem que
ter o fator 5 no denominador.
Daniel Pini wrote:
  
  
 
  
 

  Quais os termos da soma 1/2 + 1/4 + 1/6
+ 1/8 +  1/10 + 1/12 que devem ser removidos para que a soma dos termos remanecentes
seja  igual a 1?
 
  R: 1/8 e 1/10






Re: [obm-l] geometria analítica

2003-03-11 Thread A. C. Morgado




Uma soluçao sem derivadas para o problema 2:
 x^2 + y^2 + 2x - 4y +1=0 
(x+1)^2 + (y-2)^2 = 4
O centro eh C ( - 1,  2)  e o raio vale 2.
CP (raio) eh uma reta vertical (C e P tem a mesma abscissa). Logo, a tangente
eh horizontal. 
A reta horizontal por  ( - 1,  4)  eh  y = 4.
[EMAIL PROTECTED] wrote:
Olá Morgado, 
 
Como resolver estas: 
 
 
(FUVEST) A reta y= mx (m>0) é tangente à circunferência (x-4)^2 + y^2=4.
Determine o seno do ângulo que a reta forma com o eixo x. 
 
resp: 1/2 
 
(U.E. Londrina) Sejam a circunferência (lambda) x^2 + y^2 + 2x - 4y +1=0
e o ponto P(-1,4) pertencente a lambda. A equação da reta tangente lambda
pelo ponto P é: 
 
resp: y=4 
  





Re: [obm-l] problemas

2003-03-13 Thread A. C. Morgado




2) Sugeriram-me a seguinte resoluçao:
A soma dos pontos das duas equipes eh 1+2+...+10 = 55. Quem fizer 27 pontos
ou menos ganha. Logo, os escores ganhadores sao 27, 26,...,15 (15=1+2+3+4+5
eh o menor escore possivel!). Logo, ha 13 escores ganhadores.
2') Aproveito para propor um outro problema: a soluçao acima estah correta?

1) Parece-me que a melhor maneira de pensar eh com velocidade relativa. Imagine
parados os carros da outra pista e voce viajando a 120km/h (NAO AGUENTO ESSES
SIMBOLOS DE QUILOMETROS COM K MAIUSCULO!). Em 5 minutos voce percorreu 10km
e cruzou com 20 veiculos. Como eles estao igualmente espaçados, em 50 minutos
voce percorreria 100km e cruzaria com 200 veiculos. A resposta parece ser
200. Na verdade, a pergunta nao esta bem formulada. Acho que a pergunta deveria
ser      qual dos números abaixo é o mais próximo  do
número de veiculos existentes num trecho de 100 km  :-[ da pista oposta
da estrada?

Daniel Pini wrote:
  
  
 
  
 

  Numa auto estrada, o tráfego se move a uma velocidade
 constante de 60Km/h em ambas as direções. Um motorista que viaja numa das
 direções cruza 20 veiculos viajando na direção oposta em cada intervalo
de tempo  de 5 minutos. Supondo que os veiculos que trafegam na direção oposta
ao do  motorista estejam igualmente espaçados, qual dos números abaixo é
o mais próximo  do número de veiculos existentes num trecho de 100 Km de
estrada? a-100 b-120  C-200 d-240 e-400
 
   
 
  Um grupo de 10 atletas é dividido em duas equipes,
de 5  atletas cada, para disputarem um corrida rustica. O atleta que termina
a corrida  na n-ésima posição contribui com n pontos para a sua equipe. A
equipe que tiver  o menor número de pontos é a vencedora. Se não existem
empates entre os atletas,  quantos são os possíveis  escores vencedores?
 R:13






Re: [obm-l] problemas

2003-03-14 Thread A. C. Morgado




Negativo. A suposição do enunciado é de espaçamento igual na outra pista!
O enunciado (mal feito) de modo nenhum impede que o nosso motorista seja
o único em sua pista.
Minha observação é duplamente necessária: para resolver o problema e para
tentar tornar os companheiros mais críticos em relação a esses enunciados
de questões de vestibular que são, quase sempre, péssimos. Dê uma olhada
nos enunciados das questões que o Fael tem mandado, dê uma olhada nas questões
da UERJ, etc.
Ah, dirá você, é claro que ele quis dizer... Tudo bem, até acho que ele quis
dizer, mas o fato é que não disse. 
Além disso, se for para raciocinar como você (e, justiça se faça, se eu estivesse
fazendo essa prova, pensaria como você diante da omissão do enunciado) a
resposta que voce deve marcar é 400 (200 em cada pista). Eu erraria essa
questao na prova.
Veja o absurdo: voce acertaria a questao por ter errado. 
Abraços e votos de que você seja um pouco mais exigente.
Morgado

Afemano wrote:

  
  
  
 
  
 
  Ele quer saber quantos existem num trecho
de 100km,  como , dos dois lados, os carros estão com a mesma velocidade,
supoe-se que há  mesmo número de carros nas duas pistas numa mesma distância.
Ou seja, não é  necesssária essa sua observação
 
   
 
  abraços ,Gabriel
 
   
  
-
Original Message - 
   
From:
   A. C.Morgado 
   
To:
[EMAIL PROTECTED]

   
Sent:
Thursday, March 13, 2003 10:14PM
   
Subject:
Re: [obm-l] problemas
   


2) Sugeriram-me a seguinte resoluçao:
A soma dos pontos dasduas equipes eh 1+2+...+10 = 55. Quem fizer 27 pontos
ou menos ganha. Logo, osescores ganhadores sao 27, 26,...,15 (15=1+2+3+4+5
eh o menor escorepossivel!). Logo, ha 13 escores ganhadores.
2') Aproveito para propor umoutro problema: a soluçao acima estah correta?

1) Parece-me que amelhor maneira de pensar eh com velocidade relativa.
Imagine parados os carrosda outra pista e voce viajando a 120km/h (NAO
AGUENTO ESSES SIMBOLOS DEQUILOMETROS COM K MAIUSCULO!). Em 5 minutos
voce percorreu 10km e cruzou com20 veiculos. Como eles estao igualmente
espaçados, em 50 minutos vocepercorreria 100km e cruzaria com 200 veiculos.
A resposta parece ser 200. Naverdade, a pergunta nao esta bem formulada.
Acho que a pergunta deveria ser    qual dos números
abaixo é o mais próximo donúmero de veiculos existentes num trecho de
100 km :-[ da pista
oposta daestrada?

Daniel Pini wrote:
   
 
  
 

  Numa auto estrada, o tráfego se move a uma
velocidade  constante de 60Km/h em ambas as direções. Um motorista que
viaja numa das  direções cruza 20 veiculos viajando na direção oposta
em cada intervalo de  tempo de 5 minutos. Supondo que os veiculos que
trafegam na direção oposta  ao do motorista estejam igualmente espaçados,
qual dos números abaixo é o  mais próximo do número de veiculos existentes
num trecho de 100 Km de  estrada? a-100 b-120 C-200
d-240 e-400
 
   
 
  Um grupo de 10 atletas é dividido em duas equipes,
de  5 atletas cada, para disputarem um corrida rustica. O atleta que
termina a  corrida na n-ésima posição contribui com n pontos para a sua
equipe. A  equipe que tiver o menor número de pontos é a vencedora. Se
não existem  empates entre os atletas, quantos são os possíveis  escores
vencedores?  R:13


  






Re: [obm-l] Limite basico

2003-03-14 Thread A. C. Morgado
H(2n) - H(n - 1) = 1/n + 1/(n+1) + 1/(n+2) +...+ 1/(n+n) =
= 1/n  * [ 1/ ( 1+ 1/n)  +  1/ (1+ 2/n) +... + 1/ (1 + n/n) ]  cujo 
limite eh
Integral de 0 a 1 de  dx/ (1+x)  que vale  ln 2.

[EMAIL PROTECTED] wrote:

Quem sabe demonstrar que o limite de H(2n)-H(n-1) e log 2 se n e arbitrariamente
grande e H e a serie harmonica?Perdi essa demonstraçao ha algum tempo.Tem
o da Eureka 15 que to tentando fazer mas nada saiu alem do braço.Quem manja
de series de cotangente e por ai vai?
somatorio de n=1 ate infinito de arccos (1-2*(4n^4+1)^(-1))=pi/2
O primeiro e essencial,o segundo nem tanto.

TEA WITH ME THAT I BOOK YOUR FACE

--
Use o melhor sistema de busca da Internet
Radar UOL - http://www.radaruol.com.br


=
Instruções para entrar na lista, sair da lista e usar a lista em
http://www.mat.puc-rio.br/~nicolau/olimp/obm-l.html
O administrador desta lista é <[EMAIL PROTECTED]>
=
 



=
Instruções para entrar na lista, sair da lista e usar a lista em
http://www.mat.puc-rio.br/~nicolau/olimp/obm-l.html
O administrador desta lista é <[EMAIL PROTECTED]>
=


Re: [obm-l] Limite basico

2003-03-14 Thread A. C. Morgado
1) Voce pode usar tambem que H(n) = ln n + constante de Euler-Mascheroni 
+ a(n) onde a(n) tende a zero quando n tende a infinito.

[EMAIL PROTECTED] wrote:

Quem sabe demonstrar que o limite de H(2n)-H(n-1) e log 2 se n e arbitrariamente
grande e H e a serie harmonica?Perdi essa demonstraçao ha algum tempo.Tem
o da Eureka 15 que to tentando fazer mas nada saiu alem do braço.Quem manja
de series de cotangente e por ai vai?
somatorio de n=1 ate infinito de arccos (1-2*(4n^4+1)^(-1))=pi/2
O primeiro e essencial,o segundo nem tanto.

TEA WITH ME THAT I BOOK YOUR FACE

--
Use o melhor sistema de busca da Internet
Radar UOL - http://www.radaruol.com.br


=
Instruções para entrar na lista, sair da lista e usar a lista em
http://www.mat.puc-rio.br/~nicolau/olimp/obm-l.html
O administrador desta lista é <[EMAIL PROTECTED]>
=
 



=
Instruções para entrar na lista, sair da lista e usar a lista em
http://www.mat.puc-rio.br/~nicolau/olimp/obm-l.html
O administrador desta lista é <[EMAIL PROTECTED]>
=


Re: [obm-l] Termo geral da serie de Fibonacci

2003-03-14 Thread A. C. Morgado




Basta provar que vale para n=1 e n=2 e que a(n) + a(n+1) = a(n+2) para todo
n inteiro e  positivo. 
Os dois primeiros itens voce ja fez. O terceiro tem umas continhas mas da
pra fazer. Se voce acha Trigonometria complicado, faça as contas com exponenciais:
2cosz = e^z + e^(-z)
2isenz = e^z - e^(-z). 
Coragem. Em qualquer curso de Calculo, na parte de integral, todo mundo ja
fez contas piores.

felipe mendona wrote:

  
   
     Ola colegas de lista... 
 
     
 
     
 
     
 
    Segundo um colega meu,a expressao que representa o termo
geral da serie de Fibonacci(1,1,2,3,5...) é 
 
    a(n)=(sqrt5)/5.[(2.cos36)^n 
-  (2-2.cos36)^n]. 
 
      
 
       Agora eu pergunto...COMO DEMONSTRAR ISSO?
  
 
   
 
   Empiricamente pude comprovar que para n=(1,2,3) , a expressao é valida.
 
    
 
   Uma coisa eu sei , usando recorrencia talvez torne mais
pratica a demonstraçao,pois A(z) + A(z+1) = A(z+2) para todo z inteiro e 
positivo. 
 
   
 
     Abraço 
 
   Felipe Mendonça  
  
 
    Vitória-ES.
  
  
  
  MSN Hotmail, o maior webmail do Brasil.  Faça o seu agora.  2 months FREE*
= 
Instruções para entrar na lista, sair da lista e usar a lista em http://www.mat.puc-rio.br/~nicolau/olimp/obm-l.html 
O administrador desta lista é  = 
  





[obm-l] Re: [obm-l] Termo geral da serie de Fibonacci (retificaçao)

2003-03-14 Thread A. C. Morgado




Retificando o e-mail anterior. Parece que voce ja provou tambem a terceira
parte. Entao, acabou!
Por que acabou? Se fosse falso para algum valor, haveria um n minimo para
o qual eh falso. Esse n minimo nao eh nem 1 nem 2. Logo eh maior que 2. Mas
entao n-1 e n-2 sao inteiros positivos para os quais a afirmaçao eh verdadeira.
Mas pela terceira parte, se eh verdade para n-1 e n-2, tambem eh para n.,
contradiçao!

felipe mendona wrote:

  
   
     Ola colegas de lista... 
 
     
 
     
 
     
 
    Segundo um colega meu,a expressao que representa o termo
geral da serie de Fibonacci(1,1,2,3,5...) é 
 
    a(n)=(sqrt5)/5.[(2.cos36)^n 
-  (2-2.cos36)^n]. 
 
      
 
       Agora eu pergunto...COMO DEMONSTRAR ISSO?
  
 
   
 
   Empiricamente pude comprovar que para n=(1,2,3) , a expressao é valida.
 
    
 
   Uma coisa eu sei , usando recorrencia talvez torne mais
pratica a demonstraçao,pois A(z) + A(z+1) = A(z+2) para todo z inteiro e 
positivo. 
 
   
 
     Abraço 
 
   Felipe Mendonça  
  
 
    Vitória-ES.
  
  
  
  MSN Hotmail, o maior webmail do Brasil.  Faça o seu agora.  2 months FREE*
= 
Instruções para entrar na lista, sair da lista e usar a lista em http://www.mat.puc-rio.br/~nicolau/olimp/obm-l.html 
O administrador desta lista é  = 
  





  1   2   3   >